You are on page 1of 74

Universidad La Salle.

Facultad Mexicana de Medicina.


Curso de Extensin Universitaria para la Preparacin del Examen Nacional para
Aspirantes a Residencias Mdicas.

Examen de Medicina Interna 2016.

1.- Masculino de 73 aos. Llega al Servicio de urgencias por alteracin del estado de alerta, a
su ingreso se realiza glucosa capilar con resultado de 750 mg/dL. Antecedente de hipertensin
arterial desde hace 10 aos. EF desorientado, deshidratado, TA 90/60, FC 110, peso 68 kg,
estatura 1.72 cm, se coloca sonda Foley y se obtienen 40 ml de orina turbia. Laboratorio:
glucosa 810mg/dL, urea 44 mg/dL, creatinina 2 mg/dL, Na 155 mEq/L, K 3.2 mEq/L
El diagnstico del paciente es:

a) Cetoacidosis
b) Hipoglucemia
c) Estado hiperosmolar hiperglucmico
d) Estado hiperosmolar hipernatrmico

El paciente prototpico en estado hiperosmolar hiperglucmico (HHS) es un anciano con DM de


tipo 2 que tiene antecedentes de varias semanas de duracin con poliuria, prdida de peso y
decremento de la ingestin oral que culminan en confusin mental, letargo o coma. Los datos
de la exploracin fsica reflejan deshidratacin profunda e hiperosmolalidad y revelan
hipotensin, taquicardia y trastorno del estado mental. Es notable la ausencia de sntomas
como nuseas, vmitos y dolor abdominal, as como de la respiracin de Kussmaul
caracterstica de la DKA. Con frecuencia el HHS es precipitado por una enfermedad
concurrente grave, como infarto del miocardio o accidente vascular cerebral. Otros factores
precipitantes frecuentes son sepsis, neumona y otras infecciones, y es indispensable
investigar su presencia. Asimismo, pueden contribuir tambin al desarrollo de este trastorno
padecimientos debilitantes (accidente vascular cerebral previo o demencia) y situaciones
sociales que obstaculizan la ingestin de agua.

Fisiopatologa
El dficit relativo de insulina y el aporte insuficiente de lquidos son las causas que subyacen al
HHS. El dficit de insulina aumenta la produccin heptica de glucosa por el msculo
esqueltico (vase lo tratado anteriormente en la DKA). La hiperglucemia induce una diuresis
osmtica que provoca disminucin del volumen intravascular, que se exacerba todava ms por
el aporte insuficiente de lquidos. No se comprende por completo la ausencia de cetosis en el
HHS. Probablemente, el dficit insulnico es slo relativo y menos grave que en el caso de la
DKA. En algunos estudios se han encontrado concentraciones ms bajas de hormonas
contrarreguladoras y de cidos grasos libres en el HHS que en la DKA. Tambin es posible que
el hgado sea menos capaz de sintetizar cuerpos cetnicos, o que el cociente insulina/glucagon
no favorezca la cetognesis.
CRITERIOS DE DIAGNSTICO

KA EHH

Glucosa plasmtica >250 >600


pH arterial <7.3 >7.30
Bicarbonato <18 >15
cetonas urinarias positivas trazas
cetonas sricas positivas trazas
Osmolaridad variable >320
Angion gap >10 <12
Alteracin del alerta alerta estupor o
coma

Bibliografa:

1. Lobesio C: Sndrome hiperglucmico hiperosmolar. Texto de medicina


Intensiva. 5ta Ed. 2000; 95:1050-54.
2. Jimnez Murillo J, Barca C de la, Romero M, Montero F J. Coma hiperosmolar
en Medicina de urgencia. En: Jimnez Murillo J, Montero F J. Gua diagnstica y
protocolo de actuacin.2da. Ed. Madrid: Harcourt, 1999: 403-5.
3. Remuan C, lvarez JL: Coma Hiperosmolar. Rev Cubana Med 2001;
40(3):189-94.
4. Shoemaker LW: Critical Care Medicine. 3era Ed. Text Book of Critical Care.
Philadelphia Saunders. 2000; 71:783-85
5. Soler Morejn C: Coma hiperosmolar en: Temas Actualizados en: Rev Cubana
Med 1999; 38(3):183-7.
Villanueva V. Complicaciones agudas de la diabetes mellitas. Rev

2.- Femenino de 29 aos. Acude a consulta por hirsutismo e irregularidades menstruales.


Antecedente familiar de diabetes mellitus en madre y una hermana. Su padecimiento empez
desde la adolescencia con ritmo menstrual de 40 60 x 5 -7 das. EF: peso 87 kg, estatura
1.54 m, TA 130/80, FC 72x, acantosis nigricans en cuello y axilas, no galactorrea, permetro
abdominal de 112 cm. Vello supralabial, en patilla, en lnea media infraumbilical y en cara
interna de muslos. Laboratorio: glucosa 104 mg/dL, resto de qumica sangunea normal, Bh
normal. Prolactina 16 ng/dL, testosterona: 78 ng/dL (25 a 50 ng/dl). US
El diagnstico ms probable es:

a) Sndrome de ovarios poliqusticos por resistencia a la insulina


b) Hiperprolactinemia por microprolactinoma
c) Virilizacin por tumor productor de testosterona
d) Diabetes mellitus secundaria a la obesidad

El sndrome de ovarios poliqusticos (SOPQ) afecta aproximadamente a un 4% de mujeres en


edad reproductiva y se caracteriza por anovulacin crnica e hiperandrogenismo. Es la causa
ms comn de infertilidad en mujeres.
Se caracteriza clnicamente por acn, alopecia, hirsutismo, irregularidades menstruales e
infertilidad.
Los hallazgos de laboratorio ms frecuentes son: aumento de la hormona luteinizante (LH),
aumento de la relacin LH/FSH (hormona folculoestimulante), aumento de andrgenos (tanto
ovricos como adrenales) y de estrgenos circulantes. Otros hallazgos de laboratorio
habituales son una prueba tolerancia oral a la glucosa anormal y alteraciones en el perfil
lipdico.
Todo esto junto con las imgenes ecocardiogrficas caractersticas que definen al sndrome.
La teraputica permite dos grandes enfoques que pueden superponerse: la correccin de las
manifestaciones de hiperandrogenismo y el tratamiento de las alteraciones del eje reproductivo
(anovulacin, esterilidad).
Los antiandrgenos estn fundamentalmente indicados para tratar los sntomas virilizantes.
Las alternativas para inducir la ovulacin son numerosas: al citrato de clomifeno y a la antigua
reseccin en cua se agregan las gonadotrofinas humanas, pulsos de GnRH (hormona
liberadora de gonadotrofinas), medidas o frmacos para modificar los niveles de insulina, y
finalmente tcnicas quirrgicas endoscpicas para reducir la masa ovrica.

Revista de Posgrado de la VIa Ctedra de Medicina - N 125 Marzo 2003


Pg. 37-40
SINDROME DE OVARIOS POLIQUISTICOS
Dra. Sandra Beneyto, Dra. Maria Andrea Ferreyra, Dr. Andrs Galfrascoli,
Dr. Andrs Gonzlez, Dra. Susana Sosa

3.- Femenino de 24 aos que acude a consulta por prdida de peso, diarrea, palpitaciones,
temblor y labilidad emocional. No tiene antecedentes importantes. Refiere evacuaciones
diarreicas de 5 a 7 por da, sin moco o sangre.
EF: peso 52 kg, estatura 1.59, FC 108x, TA: 120/65, retraccin palpebral, hiperemia conjuntival
y de carncula, cuello con tiroides aumentada de tamao 3 veces de forma difusa,
extremidades superiores con hiperhidrosis palmar, temblor fino distal, reflejos exaltados
El diagnstico probable es:

a) Parasitosis del tipo de la giardiasis


b) Enfermedad de Graves
c) Bocio multinodular txico
d) Taquicardia paroxstica

El bocio txico difuso (BTD) constituye la forma ms frecuente de hiperfuncin de la glndula


tiroidea (70 % de los casos), que puede aparecer a cualquier edad, aunque por lo general
aparece entre la tercera y cuarta dcada de la vida. Esta enfermedad es ms frecuente en la
mujer, donde se observa un predominio de 7:1 en relacin con los hombres en regiones no
bocigenas. Esta relacin se reduce en las zonas de bocio endmico. Los factores genticos
desempean un papel esencial en la etiologa y existe una predisposicin familiar a esta
enfermedad de
Graves-Basedow.
El BTD se caracteriza por la presencia de hipertiroidismo, bocio difuso y elstico, oftalmopata,
dermopata, acropaquia tiroidea y onicolisis. Es importante el diagnstico y tratamiento precoz
del hipertiroidismo para evitar complicaciones, principalmente las cardiovasculares.
Criterios diagnsticos y teraputicos
Sospecha clnica
Sntomas de mayor valor diagnstico
Intolerancia al calor
Palpitaciones
Nerviosismo
Insomnio
Irritabilidad
Hiperquinesia
Fatigabilidad fcil
Polifagia
Prdida de peso
Hiperdefecacin y/o diarreas
Signos de mayor valor diagnstico
Bocio
Taquicardia
Temblor de las manos
Debilidad muscular
Piel hmeda y caliente
Hiperquinesia
Pelo fino y frgil

Bibliografa:

1. Bryer-Ash M. Evaluation of the patient with a suspected thyroid disorder. Obstet Gynecol Clin
North Am 2001
Jun;28(2):421-38
2. Weber AL, Randolph G, Aksoy FG. The thyroid and parathyroid glands. CT and MR imaging
and correlation
with pathology and clinical findings. Radiol Clin North Am 2000 Sep;38(5):1105-29
3. Jarlov AE, Nygaard B, Hegedus L, Hartling SG, Hansen JM. Observer variation in the clinical
and laboratory
evaluation of patients with thyroid dysfunction and goiter. Thyroid 1998 May;8(5):393-8
4. Costa AJ. Interpreting thyroid tests. Am Fam Physician 1995 Dec;52(8):2325-30

4.-Se trata de paciente masculino de 50 aos con antecedentes de tabaquismo positivo desde
los 20 aos, acude por dolor precordial relacionado al esfuerzo, de corta duracin, de 4 meses
de evolucin y con ECG en reposo normal, el siguiente estudio de eleccin es:

a) Prueba de esfuerzo con protocolo de Bruce


b) Ecocardiograma de reposo
c) Prueba de Talio- Dipiridamol
d) Ecocardiograma con estrs farmacolgico

El propsito fundamental de la prueba es el de demostrar la existencia de isquemia miocrdica


en los subgrupos de poblacin con mayor prevalencia de cardiopata isqumica o bien en los
subgrupos en donde la prueba se efecta a manera de evaluacin del tratamiento en pacientes
ya conocidos con cardiopata isqumica. Existe otro subgrupo de poblacin en quienes este
estudio ayuda a determinar la clase funcional en la que se encuentran e incluye a pacientes
con valvulopatas o con insuficiencia cardiaca de cualquier etiologa.

Protocolos de esfuerzo.

El protocolo ms empleado es el de Bruce sobre treadmill, aunque existen otros protocolos y su


eleccin depender de las condiciones del individuo.

Los protocolos discontinuos son los que alternan periodos de esfuerzo que se intercalan con
periodos de reposo de duracin similar, se emplean en escasas circunstancias. Los protocolos
continuos son los que no interrumpen el esfuerzo una vez iniciado hasta finalizada la prueba,
permiten mejor adaptacin fsica y psicolgica y es posible adaptar la intensidad de forma
individualizada para que la prueba tenga una duracin de 6 a 12 minutos.

Los protocolos mximos son los que se suspenden debido a la sintomatologa del paciente, a
los signos registrados durante la prueba o se alcanzan valores mximos de FC y VO2. Los
protocolos submximos son los que se suspenden cuando el sujeto alcanza un nivel
determinado de carga, habitualmente el 85% de la FC mxima terica (que se encuentra
entorno a los 170 lpm).

En la prctica diaria, el nivel de carga (VO2) se expresa en forma de trabajo externo (MET
equivalentes metablicos) que corresponden a 3,5ml/kg/min de VO2, lo que permite comparar
protocolos entre s (cada protocolo dispone de frmulas para realizar el clculo de los METS),
el error que cometen en el clculo de los METS es mayor en protocolos discontinuos.

Emplear la FC como nico criterio para determinar el esfuerzo mximo es errneo, por lo que
deberan tenerse en cuenta otros criterios, como es la percepcin subjetiva por parte del
paciente mediante la escala de Borg (tabla I). Esta dificultad en la prediccin del esfuerzo
mximo es lo que limita la realizacin de pruebas submximas a la determinacin de la
condicin fsica de sujetos aparentemente sanos.

Tabla I. Escala de Percepcin del esfuerzo de Borg.


Escala de 15 grados Escala de 10 grados
Valor Percepcin Valor Percepcin
6 No se siente nada 0 Nada
7 0,5 Muy muy leve
Muy muy leve
8 1 Muy leve
9
Muy leve 2 Leve
10
11 Considerablemente leve 3 Moderada
12 4 Algo fuerte
Moderadamente dura
13 5 Fuerte o intensa
14 6
Dura Muy fuerte
15 7
16 8
Muy dura
17 9 Muy muy fuerte
18 (submxima)
Muy muy dura 10
19
20 Esfuerzo mximo
* A la izquierda la escala original de esfuerzo percibido en 15 grados (de 6 a 20) y a la
derecha la ms nueva de 10 categoras.

Tabla II. Indicaciones clsicas de la ergometra.


I. Fines diagnsticos
A. Pacientes sintomticos.
1. Dolor torcico:
a) Tpico
b) Atpico
2. Clnica de equivalentes isqumicos.
A. Pacientes asintomticos.
1. Con alteraciones en el ECG sugestivas de isquemia.
2. Con alta probabilidad de padecer Cardiopata Isqumica (paciente con mltiples
factores de riesgo)
3. Cuando convenga descartar con cierta seguridad CI.
4. Con sospecha de CI silente.
5. Sedentarios que inician programa de actividad fsica.
6. Para estudio funcional de ciertas arritmias.
II. Con fines valorativos y pronsticos.
1. Seguimiento de paciente con CI conocida.
2. Tras IAM.
3. En exmenes prelaborales o laborales.
4. De la eficacia del tratamiento:
Mdico.
Cateterismo y angioplastia.
Quirrgico.
5. Respuesta de la Tensin Arterial.
6. En valvulopatas o miocardiopatas.
7. Estudio de arritmias y trastornos de la conduccin aurculo-ventricular.
8. En cardiopatas congnitas.
Bibliografa

1. Gibbons RJ (Edit.). ACC/AHA 2002 Guideline Update of Exercise Testing. 2002


American College of Cardiology Foundation and American Heart Association ACC/AHA;
2002 [Acceso 1-4-06]. Disponible en:
2. Guidelines for cardiac exercise testing. ESC Working Group on Exercise Physiology,
Physiopathology and Electrocardiography Eur Heart J 1993; 14: 969-988.
3. Fernando Ars Aros F, Boraita A, Alegria E, Alonso AM, Bardaji A, Lamiel R el al.
Guas de prctica clnica de la Sociedad Espaola de Cardiologa en pruebas de esfuerzo.
Rev Esp Cardiol 2000; 53 (8): 1063-94
4. Chaitman B. Las pruebas de esfuerzo. En: Braunwald E, editor. Tratado de Cardiologa.
Medicina Cardiovascular. 4 ed. Madrid Mc-Graw-Hill-Interamericana de Espaa; 1993. p.
177-197.
5. Schlant RC, Friesinger GC 2nd, Leonard JJ. Clinical competence in exercise testing: a
statement for physicians from the ACP/ACC/AHA Task Force on Clinical Privileges in
Cardiology. J Am Coll Cardiol 1990; 16: 1061-5
6. Reyes Lopez de los M, Iiguez Romo A, Goicolea de Oro A, Funes Lpez B, Castro
Beiras A. El consentimiento informado en cardiologa. Rev Esp Cardiol 1998; 51: 782-796.

7. Fletcher GF, Flipse T, Malouf J, Kligfield P. Current status of ECG stress testing. Curr
Probl Cardiol. 1998 Jul; 23(7): 353-423.
8. Alegra Ezquerra E, Alijarde Guimer M, Cordo Mollar JC, Chorro Gasc FJ, Pajarn
Lpez A. Utilidad de la prueba de esfuerzo y de otros mtodos basados en el
electrocardiograma en la cardiopata isqumica crnica. Rev Esp Cardiol 1997; 50: 6-14
9. Wasserman K, Hansen JE, Sue DY, Whipp BJ, Casaburi R. Principles of exercise
testing and interpretation . 2 ed. Philadelphia: Lea & Febiger; 1994. p. 95-111.
10. American college of Sports Medicine. Guideliness for exercise testing and prescription.
5 ed. Baltimore: Williams & Wilkins; 1995.
11. Borg GA. Psychophysical bases of perceived exertion. Med Sci Sports Exerc 1982; 14:
377-381
12. Froelicher VF, Umann TM. Exercise testing: clinical applications. En: Pollock ML,
Schmidt DH, editors. Heart disease and rehabilitation. 3 ed. Champaign, IL: Human
Kinetics, 1995; p.57-79.
13. Myers J, Froelicher VF. Exercise testing. Procedures and implementation. Cardiol Clin.
1993; 11(2): 199-213.
14. Weiner DA, McCabe C, Hueter DC, Ryan TJ, Hood WB Jr. The predictive value of
anginal chest pain as an indicator of coronary disease during exercise testing. Am Heart J
1978; 96: 458-462.
5.- Durante su consulta en la unidad de Medicina Familiar recibe a un paciente de 60 aos de
edad que asegura ser hipertenso de ms de cinco aos de evolucin, sin agudizaciones
adems de ser portador de una fibrilacin auricular, motivo por los cuales recibe propafenona
150mg cada maana. Al revisar su electrocardiograma usted espera encontrar:

a) Ausencia de onda P, complejo QRS normal e irregularidad de los intervalos R-R.


b) Ausencia de onda P, complejo QRS ensanchado, intervalos R-R sin alteracin
c) Presencia de onda P, complejo QRS ensanchado y eje rotado a la derecha.
d) Ritmo nodal con disociacin A-V

Guadalajara J. Cardiologa. Sexta Edicin 151 152 La fibrilacin auricular es la arritmia


cardiaca ms frecuente. La despolarizacin catica y desordenada de las aurculas por
mltiples ondas en simultnea, trae como consecuencia que se pierda la funcin mecnica de
la contraccin auricular. Estos dipolos de activacin mltiple y desordenados alcanzan al nodo
A-V y penetran en l, algunos pasan hacia el Haz de His mientras que otros no lo alcanzan,
debido a la penetracin parcial del nodo por conduccin decreciente completa, esto se conoce
como, conduccin oculta; as, la rpida penetracin de los estmulos auriculares favorece la
aparicin de conduccin oculta, la cual afecta en forma impredecible el periodo refractario del
nodo. El trazo electrocardiogrfico tpico es la ausencia de onda P, complejo QRS normal e
intervalos R-R diferentes.

6.- Se trata de masculino de 47 aos de edad que se queja de dolor y rigidez en las
articulaciones de 3 meses de evolucin. Ambos pies y ambas manos estn calientes as como
las articulaciones edematizadas. De los siguientes quien sugiere el diagnstico de artritis
reumatoide es:

a) Adormecimiento y palidez de los dedos al ser expuestos al fro


b) Afeccin simtrica de las articulaciones distales interfalngicas
c) Lquido sinovial con alta viscosidad y 30 mil linfocitos /mm3
d) Rigidez matutina que dura 1 hr

ARTRITIS REUMATOIDE
CRITERIOS DE CLASIFICACIN 2010
DE ACR/EULAR
Criterios de clasificacin para AR (algoritmo basado en
puntaje: sumar el puntaje de las categoras A-D: se
necesita un puntaje de 6/10 para clasificar a un paciente
con AR definida
A. Involucro articular
1 articulacin grande 0
2-10 articulaciones grandes 1
1-3 articulaciones pequeas (con o sin artic grandes) 2
4-10 articulaciones pequeas (con o sin artic grandes) 3
> 10 articulaciones (por lo menos 1 articulacin pequea) 5

Aletaha D, et al. Arthritis Rheum. 2010;62:2569-81.


ARTRITIS REUMATOIDE
CRITERIOS DE CLASIFICACIN 2010
DE ACR/EULAR

B. Serologa (se necesita por lo menos una prueba positiva)


FR y anticuerpos anti-PCC negativos 0
FR bajo positivo o anticuerpos anti-PCC bajos positivos 2
FR alto positivo o anticuerpos anti-PCC altos positivos 3
C. Reactantes de fase aguda (se necesita por lo menos una prueba
positiva)
PCR y VSG normales 0
PCR o VSG anormal 1
D. Duracin de los sntomas
< 6 semanas 0
6 semanas 1

Aletaha D, et al. Arthritis Rheum. 2010;62:2569-81.

7.- Masculino de 55 aos de edad, que se queja de debilidad muscular desde hace
aproximados 3 meses. A la E.F. Se aprecia eritema en heliotropo en su trax superior, cuello
y cara. Eritema maculopapular situado sobre nudillos de ambas manos.
De los siguientes, el que se asocia al cuadro clnico de esta patologa es:

a) Debilidad muscular proximal


b) Ataxia
c) Hiperreflexia de tendones
d) Inflamacin de pequeas articulaciones

Dermatomiositis:

La polimiositis respeta la piel, mientras que la dermatomiositis presentar alteraciones


cutneas caractersticas acompaando a la afectacin muscular
Cuadro Clnico:
- Alteraciones musculares. Vienen marcadas por la presencia de debilidad muscular aguda
o subaguda (generalmente de inicio insidioso), simtrica y difusa, con preferencia por
musculatura proximal de extremidades (cintura plvica y escapular), tronco y cuello. En la
mayora de los casos es indoloro. Con el tiempo, desarrollan atrofia, contracturas y disminucin
de los reflejos.
- Alteraciones cutneas. La ms frecuente en la DM es una erupcin cutnea
eritematoviolcea que afecta a cuello, cara y trax. Es caracterstico tambin, el eritema
heliotropo (en prpados), que puede extenderse a otras zonas fotoexpuestas), las ppulas
de Gottron (localizadas en los nudillos), telangiectasias periungueales, a veces ulceracin
drmica y calcinosis (fundamentalmente en la DM infantil).
- Articulares. Artralgias, artritis transitorias, no erosivas, con tendencia a la simetra.
- Otras. Afectacin cardiaca variable (alteracin ECG, arritmia, miocarditis), pulmonar (fibrosis
intersticial asociada con anti Jo-1), renal (muy rara), fenmeno de Raynaud

Diagnstico:
- Analtica: aumento de VSG y de enzimas musculares, (CPK, aldolasa, GOT, GPT, LDH). La
CPK es la ms sensible y la que guarda una mejor correlacin clnica con la actividad de la
enfermedad y la valoracin de recadas. El FR es + en 20% y
ANA es + en 10-30%. Si la destruccin muscular es intensa, puede producir mioglobinuria.
- Destacan anticuerpos: anti-Jo1: en casos de PM asociado a neumonitis intersticial (sndrome
antisintetasa-miosistis, fibrosis pulmonar, artritis no erosiva y fenmeno de Raynaud).
anti-PM1 o PM-Scl: asociacin con esclerodermia.
anti-Mi, en DM.
antimioglobina.

BIBLIOGRAFA RECOMENDADA

Klippel JH, Stone JH, Crofford LJ, White PH, editors. Primer on the rheumatic diseases.
13th ed. New York: Springer-The Arthritis Foundation; 2008.

Martnez-Elizondo P, editor. Introduccin a la Reumatologa. 4a ed. Mxico: Colegio Mexicano


de Reumatologa A.C./Intersistemas S.A. de C.V.; 2008.

Firestein GS, Budd RC, Harris ED Jr, McInnes IB, Ruddy S, Sergent JS, editors. Kelleys
Textbook of Rheumatology. 8th ed. Philadelphia: Saunders Elsevier; 2009.

8.- Masculino de 64 aos presenta dolor, calor y tumefaccin en la rodilla derecha. De los
siguientes hallazgos el ms til para establecer el diagnstico de pseudogota en este paciente
es:

a) Crecimiento de articulaciones interfalngicas proximales e interfalngicas distales


b) Calcio en el menisco en una radiografa de la rodilla afectada
c) Altas concentraciones sricas de cido rico
d) Cristales con birrefringencia negativa del lquido de la rodilla

Allen R. M. MMS Medicina Interna. 5. Edicin. National Medical Series. Mc. Graw Hill. 2006.
(Captulo 10 IV B 5 b). El hallazgo de un menisco calcificado en la radiografa de la rodilla
afectada es un dato diagnstico de enfermedad por dihidrato de pirofosfato de calcio, que
sugiere que la inflamacin de la rodilla es causada por seudogota. El crecimiento de las
articulaciones interfalngicas proximales y distales sugiere slo osteoartritis y no una causa
especfica. El aumento srico de urato se relaciona con gota. El aumento de cristales con
birrefringencia en examen de luz polarizada compensada roja, de lquido sinovial, es especfico
para el diagnstico de seudogota. Hay muchas causas de derrame inflamatorio aparte de esta
entidad.

9.- Se trata de femenino de 41 aos de edad refiere ser sana, experimenta dolor retroesternal
sbito con fiebre y falta de aire. Es fumadora y no toma medicamentos excepto anticonceptivos
orales. En la exploracin fsica se encuentran taquipnea y temperatura de 38C. Los datos de
auscultacin, percusin y radiogrficos del trax son normales. De los siguientes diagnsticos
el ms probable es:

a) Traqueobronquitis
b) Neumona atpica
c) Embolia pulmonar
d) Neumona bacteriana

Allen R. M. MMS Medicina Interna. 5. Edicin. National Medical Series. Mc. Graw Hill. 2006.
(captulo 2 VIII E 1, 2 a; captulo 8 V C 3, 4). El diagnstico ms probable es embolia pulmonar.
El inicio agudo descarta neumona atpica y hace poco probable el cncer pulmonar. Sin
pruebas de tos productiva es poco probable que haya traqueobronquitis, trastorno tambin
subagudo. La neumona bacteriana es muy improbable junto con la radiografa de trax. El
tabaquismo y el uso de anticonceptivos orales predisponen a trombosis venosa profunda y
embolias pulmonares.

10.- Se trata de masculino de 35 aos que acude por fiebre de comienzo sbito, tos con
expectoracin purulenta y dolor en el hemitrax que aumenta al respirar. La placa de trax
muestra signos de consolidacin pulmonar y un infiltrado lobular. La sospecha diagnstica ms
probable del agente causal y tratamiento ambulatorio para el mismo es:

a) Chlamydia /azitromicina
b) Coxiella /vacomicina
c) M. pneumoniae /penicilina
d) S. pneumoniae / claritromicina

En ste caso lo ms probable es que el paciente se encuentre cursando con una neumona
adquirida en la comunidad originada en el 60-70% de los casos por estreptococo pneumoniae
el tratamiento ambulatorio en la actualidad se utiliza macridos , betalactmicos,
fluoroquinolonas.

MACROLIDOS
a.- Claritromicina 500 mg VO / 12 hrs
b.- azitromicina 500 mg VO / 24 hrs
c.- Telitromicina 800 mg VO / 24 hrs

BETA LACTAMICOS
a.- Amoxicilina-Ac Clavulanico 500 mg VO / 8 hrs
AMBULATORIO 875 mg VO cada 12 hrs

QUINOLONAS
a.- Levofloxacino 500 mg VO / 24 hrs
b.- Moxifloxacino 400 mg VO / 24 hrs

1.- Fishman AP, , Fishman JA, Grippi MA, Kaisser LR, Seor RM. Pulmonary Diseases and
disorder. 3a. Edicin McGraw-Hill, EU.
2.- Fraser, R ; Neil, C; Par, P; Diseases of the Chest, Third Edition, Editorial Elsevier,
3.- Murray and Nadels; Textbook Respiratory Medicine, Vol 1-2, Elsevier editorial,.
11.- Mujer de 20 aos que acude a consulta externa, refiere que desde hace ms de un ao
presenta astenia, cansancio, prdida de apetito y dificultades para concentrarse en los
estudios. Al interrogatorio refiere que a perdido inters en los estudios, frecuenta menos a sus
amigos, con pesimismo en la mayora de sus actividades. El diagnstico ms probable es:

a) Trastorno Distmico
b) Anorexia nerviosa
c) b) Trastorno de ansiedad.
d) Depresin mayor

Criterios para el diagnstico de


F34.1 Trastorno distmico (300.4)

A. Estado de nimo crnicamente depresivo la mayor parte del da de la mayora de los das,
manifestado por el sujeto u observado por los dems, durante al menos 2 aos.

Nota: En los nios y adolescentes el estado de nimo puede ser irritable y la duracin debe ser
de al menos 1 ao.

B. Presencia, mientras est deprimido, de dos (o ms) de los siguientes sntomas:

1. perdida o aumento de apetito


2. Insomnio o hipersomnia
3. Falta de energa o fatiga
4. Baja autoestima
5. Dificultades para concentrarse o para tomar decisiones
6. Sentimientos de desesperanza

C. Durante el perodo de 2 aos (1 ao en nios y adolescentes) de la alteracin, el sujeto no


ha estado sin sntomas de los Criterios A y B durante ms de 2 meses seguidos.

D. No ha habido ningn episodio depresivo mayor durante los primeros 2 aos de la alteracin
(1 ao para nios y adolescentes); por ejemplo, la alteracin no se explica mejor por la
presencia de un trastorno depresivo mayor crnico o un trastorno depresivo mayor, en remisin
parcial.

Nota: Antes de la aparicin del episodio distmico pudo haber un episodio depresivo mayor
previo que ha remitido totalmente (ningn signo o sntoma significativos durante 2 meses).
Adems, tras los primeros 2 aos (1 ao en nios y adolescentes) de trastorno distmico,
puede haber episodios de trastorno depresivo mayor superpuestos, en cuyo caso cabe realizar
ambos diagnsticos si se cumplen los criterios para un episodio depresivo mayor.

E. Nunca ha habido un episodio manaco, un episodio mixto o un episodio hipomanaco y


nunca se han cumplido los criterios para el trastorno ciclotmco.

F. La alteracin no aparece exclusivamente en el transcurso de un trastorno psictico crnico,


como son la esquizofrenia o el trastorno delirante.

G. Los sntomas no son debidos a los efectos fisiolgicos directos de una sustancia (p. ej., una
droga, un medicamento) o a enfermedad mdica (p. ej., hipotiroidismo).

H. Los sntomas causan un malestar clnicamente significativo o deterioro social, laboral o de


otras reas importantes de la actividad del individuo.

Especificar si:
Inicio temprano: si el inicio es antes de los 21 aos
Inicio tardo: si el inicio se produce a los 21 aos o con posterioridad

Especificar (para los ltimos 2 aos del Trastorno distmico):

Con sntomas atpicos

DSM IV

MANUAL DIAGNSTICO Y ESTADSTICO DE LOS TRASTORNOS MENTALES


(American Psychiatric Association)

12.- Masculino de 64 aos, jubilado es ingresado al servicio de urgencias. Los familiares


refieren, que el da de ayer de manera brusca el paciente inici con confusin, comenz a decir
cosas raras, a no responder a lo que le preguntaban, incluso present confusin de su propio
nombre. Al interrogatorio se muestra distrado y parece no entender lo que se le pregunta, no
recuerda nada de lo que le ha pasado y no sabe ni el da, ni el lugar en el que est. El
diagnstico ms probable de ste paciente es:

a) b). Psicosis breve.


b) Alzheimer.
c) Delirium
d) Esquizofrenia.

MANIFESTACIONES CLNICAS
Los hallazgos cardinales del delirium incluyen su aparicin aguda y la inatencin. Para definir el
momento exacto del inicio es necesario recurrir a la informacin dada por el cuidador del
paciente. Otra de las caractersticas es la fluctuacin que presenta el cuadro a lo largo del da.
El individuo fcilmente se distrae ante los diferentes estmulos externos, y es muy difcil que
sostenga una conversacin y obedezca rdenes. Aunado a lo anterior puede presentarse
desorganizacin del pensamiento, alteraciones de la sensopercepcin y malteracin del estado
de conciencia (generalmente letrgico). Aunque no son sntomas principales, tambin
presentan desorientacin, dficit cognoscitivo, agitacin o retardo psicomotriz, delirios, labilidad
emocional e inversin del ciclo sueo-vigilia.

Criterios para el diagnstico de


F05.0 Delirium debido a... (Indicar enfermedad mdica) (293.0)

A. Alteracin de la conciencia (p. ej., disminucin de la capacidad de atencin al entorno) con


disminucin de la capacidad para centrar, mantener o dirigir la atencin.

B. Cambio en las funciones cognoscitivas (como dficit de memoria, desorientacin, alteracin


del lenguaje) o presencia de una alteracin perceptiva que no se explica por la existencia de
una demencia previa o en desarrollo.

C. La alteracin se presenta en un corto perodo de tiempo (habitualmente en horas o das) y


tiende a fluctuar a lo largo del da.

D. Demostracin a travs de la historia, de la exploracin fsica y de las pruebas de laboratorio


de que la alteracin es un efecto fisiolgico directo de una enfermedad mdica.
Nota de codificacin: Si el delirium est superpuesto a una demencia vascular preexistente,
indicarlo codificando F01.8 Demencia vascular, con delirium [290.41].

Nota de codificacin: Incluir el nombre de la enfermedad mdica en el Eje I, por ejemplo,


F05.0 Delirium debido a encefalopata heptica [293.0]; codificar tambin la enfermedad
mdica en el Eje III.

13.- Acude al servicio de urgencias masculino de 34 aos acompaado por su esposa agitado
e inquieto porque dice que le persiguen unos asesinos que van a matarle. Se realiza
exploracin fsica observndose pupilas midriticas, temperatura de 37,9C, Fc 110 lpm y TA
de 155/95 mmHg, sin otros patolgicos aparentes. Su familiar afirma que tiene historia de
abuso de drogas. La droga que ms probablemente ha producido esta reaccin es:

a) Heroina
c) Diacepam
d) Alcohol
b) Cocana

Los criterios internacionales de diagnstico de acuerdo al


DSM-IV TR son los que a continuacin se detallan:

A. Consumo reciente de cocana.

B. Cambios psicolgicos comportamentales desadaptativos clnicamente significativos


(sexualidad inapropiada, comportamiento agresivo, labilidad emocional, deterioro de la
capacidad de juicio y deterioro de la actividad laboral o social) que se presentan durante la
intoxicacin o pocos minutos despus del consumo de cocana.

C. Dos o ms de los siguientes signos, que aparecen durante o poco tiempo despus del
consumo de cocana:

(1) Taquicardia o bradicardia


(2) dilatacin pupilar
(3) aumento o disminucin de la tensin arterial
(4) sudoracin o escalofros
(5) nuseas o vmitos
(6) prdida de peso demostrable
(7) agitacin o retraso psicomotores
(8) debilidad muscular, depresin respiratoria, dolor en el pecho o arritmias cardacas
(9) confusin, crisis comiciales, discinesias, distonas o coma

D. Los sntomas no se deben a enfermedad mdica ni se explican mejor por la presencia de


otro trastorno mental.

La caracterstica esencial de intoxicacin por cocana es la presencia de cambios psicolgicos


o comportamentales desadaptativos clnicamente significativos que aparecen durante o poco
tiempo despus del consumo de cocana (Criterios A y B). La intoxicacin por cocana empieza
habitualmente con una sensacin de euforia (high) e incluye uno o ms de los sntomas
siguientes: euforia con incremento de la sensacin de vigor, sociabilidad, hiperactividad,
inquietud, hipervigilancia, sensibilidad interpersonal, charlatanera, ansiedad, tensin, estado
de alerta, grandiosidad, comportamientos estereotipados y repetitivos, rabia o clera y deterioro
de la capacidad de juicio y, en el caso de intoxicacin crnica, afectividad embotada, cansancio
o tristeza y retraimiento social. Estos cambios psicolgicos y comportamentales se acompaan
de dos o ms de los siguientes signos y sntomas, que aparecen durante la intoxicacin o poco
tiempo despus: taquicardia o bradicardia, dilatacin pupilar, aumento o disminucin de la
tensin arterial, sudoracin o escalofros, nuseas o vmitos, prdida de peso demostrable,
agitacin o retraso psicomotores, debilidad muscular, depresin respiratoria, dolor en el pecho
o arritmias cardacas y confusin, crisis comiciales, discinesias, distonas o coma (Criterio C).
La intoxicacin aguda o crnica se asocia a menudo con deterioro de la actividad social o
laboral. La intoxicacin grave puede conducir al coma. Para establecer el diagnstico de
intoxicacin por cocana los sntomas no han de ser debidos a enfermedad mdica ni
explicarse mejor por la presencia de otro trastorno mental (Criterio D).
La magnitud y el tipo de los cambios psicolgicos o comportamentales dependen de muchas
variables, que incluyen la dosis consumida y las caractersticas individuales del sujeto que
consume la sustancia (p.ej., tolerancia, grado de absorcin, cronicidad del consumo y contexto
en el que se ingiere la droga). Los efectos estimulantes observados ms frecuentemente son
euforia, aumento del pulso y la tensin arterial, y actividad psicomotora. Los efectos depresores
como tristeza, bradicardia, descenso de la tensin arterial y disminucin de la actividad
psicomotora son menos frecuentes y slo aparecen con el consumo crnico de dosis altas.

14.- Zumpango es una comunidad de 100,000 personas. Durante 2008 hubo 1,000 defunciones
por todas las causas. Durante el mismo ao se registraron un total de 300 casos de Infartos
Agudos al Miocardio y 60 defunciones por esta misma causa.
La tasa de mortalidad bruta en Zumpango es:
a) 100 por 1,000
b) 10 por 1,000
c) 300 por 1,000
d) 150 por 1,000

La tasa de mortalidad bruta mide la proporcin de la poblacin que muere cada ao o el


nmero de muertes en la comunidad por 1, 000 habitantes. Se calcula dividiendo todas las
muertes de un ao entre la poblacin total por 1, 000. ( TM= 1,000 / 100, 000 por 1, 000)

Morton R. F. Bioestadstica y Epidemiologa, Interamericana, 3. Ed. 1993; pg: 19

15.- Para valorar que tan fuerte es la asociacin entre una exposicin y una enfermedad, cul
sera la medida de asociacin a calcular?

a) Riesgo relativo
b) Tasa de mortalidad
c) Incidencia
d) Prevalencia

El Riesgo Relativo mide la fortaleza de una asociacin entre un factor y un cierto resultado
final; de este modo, un Riesgo Relativo orienta hacia causalidad y es til para investigar el
origen de una enfermedad,

Morton R. F. Bioestadstica y Epidemiologa, Interamericana, 3. Ed. 1993; pg: 36


16.- El riesgo de adquirir infeccin por virus del papiloma humano entre estudiantes
universitarios se mide en un estudio epidemiolgico a travs de:

b) La densidad de incidencia
c) La tasa de incidencia dividida entre la prevalencia
d) La incidencia acumulada
e) Prevalencia

La incidencia de una enfermedad se mide de dos formas: mediante la densidad de incidencia


que expresa la ocurrencia de la enfermedad entre la poblacin en relacin con unidades de
tiempo-persona, por lo que mide la velocidad de ocurrencia de la enfermedad y la incidencia
acumulada que expresa el volumen de casos nuevos ocurridos en una poblacin durante un
periodo, y mide la probabilidad o riesgo de los miembros de una poblacin, de contraer una
enfermedad en un periodo especfico.

Moreno A. Principales medidas en epidemiologa. Rev Salud Pblica Mex, 2000;42(4): 343

17.- A 70-year-old man with a history of chronic obstructive pulmonary disease (COPD)
presents complaining of worsening shortness of breath for the last several days. He is coughing
large amounts of yellow-colored sputum and is no receiving no relief from his - agonist and
ipratropium aerosolized pumps. On physical examination, the patients respiratory rate is 40/min
and his heart rate is 110/min. His blood pressure is 150/85 mmHg. The patient is afrebrile. He is
using his accessory muscles of respiration to assist in breathing. Lung examination reveals
inspiratory and expiratory diffuse wheezing. Which of the following is the most likely diagnosis?

a) Chronic bronchitis
b) Exacerbation of asthma
c) Pneumonia
d) Acute exacerbation of COPD

La definicin de exacerbacin aguda de EPOC es difusa y frecuentemente se explica como la


combinacin de: empeoramiento de la disnea, aumento en la purulencia del esputo (cambio de
color) y aumento del volumen del esputo. En estos episodios se presenta una prdida
transitoria de la funcin pulmonar que, usualmente, se restaura en los siguientes meses.
Se estima que un paciente con
EPOC presenta de uno a cuatro episodios de exacerbacin aguda al ao.
Los intervalos entre las exacerbaciones son inversamente proporcionales a la progresin de la
enfermedad.
La mayora de los pacientes con EPOC compensada presentan tos que, usualmente, no
interfiere con la calidad de vida. La presencia de tos diaria, crnica, es factor pronstico de
exacerbaciones frecuentes.
No todas las exacerbaciones agudas son tratadas hospitalariamente. Se estima que cerca de
50% de las exacerbaciones no son reportadas a los servicios mdicos.
En la exacerbacin aguda, la tos es ms grave y se asocia con aumento en la produccin de
esputo y cambio de la expectoracin de mucosa a purulenta. Ocasionalmente, puede cursar
con hemoptisis que puede hacer sospechar infeccin.
Durante la exacerbacin aguda, la disnea aumenta comparada con la lnea basal del paciente
(clase funcional habitual).
El paciente puede presentar cambios mnimos en el examen fsico durante la exacerbacin
leve.
Progresivamente, mientras la exacerbacin aumenta en gravedad o la
EPOC basal es ms grave, el paciente puede presentar signos de hipoxia como aprehensin,
agitacin psicomotora, taquipnea, taquicardia, hipertensin arterial y cianosis.
Al aumentar el trabajo respiratorio, el paciente puede hacer uso de los msculos accesorios de
la respiracin y presentar diaforesis.
El cuadro puede progresar a la fatiga de los msculos respiratorios (diafragma), y es evidente
la disociacin toracoabdominal (signo ominoso de inminencia de paro respiratorio).
Finalmente, el paciente progresa a falla respiratoria, caracterizada por aumento progresivo de
la hipercapnia y la acidosis, clnicamente apreciable en las alteraciones del estado de
conciencia (somnolencia y estupor), hipopnea y apnea.

Bibliografa:

American Thoracic Society and European Respiratory Society: Standards for de diagnosis and
management pf patients with COPD. 2004 ATS/ERS Task Force.Standards for the diagnosis
and treatment of patients with COPD: a summary of theATS/ERS position paper. Eur Respir J.
2004 Jun;23(6):932-46 [PubMed]
Appleton S, Jones T, Poole P, Pilotto L, Adams R, Lasserson TJ, Smith B, Muhammad J.
Bromuro de ipratropio versus agonistas beta2 de accin prolongada para la enfermedad
pulmonar obstructiva crnica estable; 2006 (Revisin Cochrane traducida). En: La Biblioteca
Cochrane Plus, 2008 Nmero 1. Oxford: Update Software Ltd. [Resumen]
Carrasco Garrido P, Diez J M, Rejas Gutierrez J, Martin Centeno A, Gobartt Vazquez E,Gil de
Miguel A, Garca Carballo M and Jimnez Garca R. Negative impact of chronic obstructive
pulmonary disease on the health-related quality of life of patients. Results of the EPIDEPOC
study. Health Qual Life Outcomes. 2006 May 23;4:31 [PubMed] [Texto completo]
CKS. Chronic obstructive pulmonary disease [Internet]. National Health Service; 2007. [acceso
6/2/2009]. Disponible en: http://cks.library.nhs.uk/

18.- Thrombocytopenia that is caused by increased platelet destruction is most closely


associated with which of the following conditions?

a) Combination chemotherapy
b) Acute leucemia
c) Systemic lupus erythematosus
d) Excessive ethanol intake

El diagnstico de LEG se hace mediante la identificacin de manifestaciones clnicas de la


enfermedad acompaadas de uno o ms autoanticuerpos tpicos (Tablas 2 y 3).

Tabla 3
Criterios para la Clasificacin del Lupus Eritematoso Generalizado
Eritema malar
Eritema discoide
Fotosensibilidad
Ulceras orales
Artritis
Serositis
a. pleuritis
b. pericarditis

Renal:
c. proteinuria persistente > 0,5 g/24 h o +++
d. cilin

Neurolgicos:
e. convulsiones
f. psicosis

Hematolgicos:
g. Anemia hemoltica con reticulocitosis
h. Leucocitopenia < 4.000 por dos veces
i. Linfocitopenia < 1.500 por dos veces
j. Trombocitopenia < 100.000

10. Inmunolgicos:
a) Anti-DNA elevado
b) Anti-Sm
c) Antifosfolpidos: Anticardiolipinas (IgG o IgM) o Anticoagulante
lpico o VDRL falso positivo por 6 meses.
11. Anticuerpos antinucleares (en ausencia de lupus por drogas)

19.- Masculino de 70 aos, diagnosticado de carcinoma de prstata, en este momento acude


al servicio de urgencias por presentar confusin mental, nuseas, vmitos y estreimiento. Se
realizan pruebas de laboratorio y destaca una calcemia de 16mg/Dl. La primera medida
teraputica que debera tomar ante ste paciente es:

a) Hormonoterapia (leuprolide y estrgenos).


b) Difosfonatos por via oral.
c) Administracin de Solucin salina y furosemida por va intravenosa.
d) Glucocorticoides por va intravenosa.

Los sntomas que provoca la hipercalcemia estn en relacin con sus niveles en sangre.
Valores comprendidos entre 10,5 y 12 gr/dl no suelen provocar sntomas; a partir de estos
valores la sintomatologa es progresiva, afectando con ms intensidad y gravedad a todos los
sistemas del organismo. No est establecido un orden de aparicin de la sintomatologa, ni su
correspondencia con los valores de calcemia.

Sntomas de la hipercalcemia

Sistema nervioso central


Desrdenes mentales
Dificultades cognitivas
Ansiedad
Depresin
Confusin, estupor y coma
Calcificacin corneal
Suicidios (descritos aisladamente)
Sistema neuro-muscular
Fatiga o cansancio muscular
Mialgias
Descenso de la funcin de msculos respiratorios
Laxitud articular
Sistema renal
Nefrolitiasis
Diabetes inspida nefrognica (poliuria y polidipsia)
Deshidratacin
Nefrocalcinosis
Sistema gastrointestinal
Nuseas y vmitos
Anorexia
Estreimiento
Dolor abdominal
Pancreatitis
lcera pptica
Sistema esqueltico
Dolor seo
Artritis
Osteoporosis
Ostetis fibrosa qustica
Resorcin subperistica
Quistes seos
Embarazo
Hipoparatiroidismo neonatal
Tetania neonatal
Bajo peso al nacer
Retraso crecimiento intrauterino
Hiperemesis gravdica
Alta morbilidad neonatal y materna
Partos pretrmino
Sistema cardiovascular
Hipertensin arterial
Calcificacin vascular
Calcificacin miocrdica
Hipertrofia miocrdica
Acortamiento intervalo QT
Arritmias cardacas
Otros
Queratitis
Conjuntivitis
Anemia normoctica normocrmica
Gota o pseudogota

El tratamiento de la hipercalcemia est orientado fundamentalmente a eliminar la causa que la


produce. En los casos sintomticos se requiere de un tratamiento inicial especfico. Debe ser
tratada la que presenta sntomas o supera los 14 mg/dl. Los mecanismos para actuar sobre la
hipercalcemia son:

Aumento de la eliminacin renal de calcio.


Disminucin de la absorcin intestinal de calcio.
Disminucin de la resorcin sea de calcio.

El tratamiento siempre debe iniciarse con la rehidratacin del paciente, que produce un
aumento del volumen extracelular, alcanzando una ganancia de volumen de 1,5- 2,5 litros en
las primeras 24 horas.
A continuacin, se utilizar un diurtico de asa, tipo furosemida, que aumenta la excrecin
renal de sodio y calcio. La dosis de diurtico se ajustar en funcin de las cifras de calcio a lo
largo del tratamiento.

Cuando la funcin renal est comprometida puede recurrirse a la hemodilisis con calcio bajo
en el lquido de dilisis.

En los casos en los que est implicado un aumento de la resorcin sea como causa de
hipercalcemia, deberemos controlarla con:

Bisfosfonatos: disponemos de clodronato y pamidronato. El ms empleado es el


primero, porque la dosificacin es ms fcil. Su administracin en el caso de la
hipercalcemia es intravenosa, inicia el efecto a las 72 horas y alcanza el mximo a la
semana.
Calcitonina: acta ms rpido que los bisfosfonatos, pero slo mantiene el efecto
durante 72 horas.
Otros frmacos antirresortivos: Mitramicina y el nitrato de galio, con menor experiencia,
pueden usarse en caso de fracaso de los anteriores.

En aquellos casos en los que est aumentada la absorcin intestinal de calcio, como en la
produccin endgena de vitamina D (enfermedades granulomatosas o linfomas) deben tratarse
con glucocorticoides.

LECTURAS RECOMENDADAS

1. Achogue HJ, Madias EN. Changes in plasma potassium concentration during


acute acid base disturbances. Am J Med 71:456, 1981
2. Humphreys MH. Urgencias por trastornos en los lquidos, electrolitos y
equilibrio cido bsico. En: Diagnstico y Tratamiento de Urgencias. Editado
por MT Ho y CE Saunders. Editorial El Manual Moderno. Mxico DF, 1991
3. Mora JM, Delgado VA, D'Achiardi R. Trastornos del potasio. En: Manual de
Urgencias en Medicina Interna. Captulo Central. Ediciones Act Med Colomb.
Santaf de Bogot, 1994
4. Silva E, Lpez C, Ramrez J. Trastornos del potasio. En: Compendio de
Teraputica. Segunda edicin. Asociacin Colombiana de Medicina Interna.
Captulo Central. Ediciones Act Med Colomb. Santaf de Bogot, 1992

20.- Se trata de masculino de 29 aos, acude al servicio de Urgencias por cefalea intensa de
12 horas de evolucin. Al interrogatorio dirigido refiere debilidad muscular desde hace 4 meses.
Exploracin fsica: TA: 170/110, FC:96 x. Estatura: 1.75 cm, peso 80 kg. Laboratorio: glucosa
72 mg/dL, BUN 4.5 mg/dl, creatinina 0.72 mg/dl, Na: 142 mq/l, K: 2.1 mEq/l.
En este paciente el diagnstico probable es:

a) hipertensin secundaria a resistencia a la insulina.


b) feocromocitoma
c) hipertensin esencial.
d) hiperaldosteronismo primario

El hiperaldosteronismo primario (HAP) es una de las causas conocidas de hipertensin arterial.


En estos casos, la HTA es secundaria a una produccin excesiva y autnoma de aldosterona,
que a nivel renal induce un aumento en la reabsorcin de sal y agua, lo que se traduce en un
aumento del volumen intravascular y, secundariamente, en elevacin de la presin arterial
Adenoma
Hiperplasia adrenal bilateral idioptica
Hiperaldosteronismo suprimible
con glucocorticoides . . . . .<1%
Carcinoma. . . . . . . . . . . . . . . . . . . . . .1%

PRIMARIO
2% a 10%
DE LOS PACIENTES CON HAS
hasta 23 % de los casos
de HAS de difcil control

HIPERALDOSTERONISMO

sodio Na+

NO EDEMA
K+
H+

Alcalosis metablica
hipokalmica

21.- Masculino de 45 aos con hipercalcemia asintomtica. El resto de sus exmenes de


laboratorio muestran elevacin de parathormona, baja en fsforo, elevacin de cloro y BUN y
creatinina normales. El calcio urinario est elevado.
El diagnstico ms probable es:

a) Mileoma mltiple
b) Hiperparatiroidismo primario
c) Hipervitaminosis
d) Sarcoidosis

Al grupo de sndromes que tienen como caracterstica comn la secrecin excesiva y no


controlada de hormona para-tohormona (HPT) por una o ms paratiroides funcionantes se le
conoce como hiperparatiroidismo primario (HPP). La causa ms frecuente es el adenoma y le
sigue en frecuencia la hiperplasia.
El cuadro clnico del HPP afecta principalmente al sistema seo, al rin y al sistema
gastrointestinal. El cuadro clnico se caracteriza por debilidad muscular, fatigabilidad fcil,
manifestaciones gastrointestinales y depresin. Cuando hay afeccin renal el paciente presenta
hipercalciuria (25%), litiasis renal recidivante (25%) y/o nefrocalcinosis (20%). Cuando hay
resorcin subperistica, tumores pardos en huesos largos o en el maxilar inferior y a nivel del
crneo, las lesiones osteolticas le dan el aspecto de sal y pimienta. En los exmenes de
laboratorio hay hipercalcemia, hipofosfatemia, fosfatasa alcalina elevada (fraccin sea),
elevacin de HPT (90%), osteocalcina y desoxipiridinolinas; as como calciuria e hiperfosfaturia.
Cuando hay afeccin del sistema gastrointestinal hay lceras y hemorragias.
Por lo tanto, como podemos observar, se trata de una paciente con HPP clsico con afeccin
del sistema seo y del rin, sin manifestaciones gastrointestinales aparentemente, el cual al
someterse a tratamiento quirrgico y mdico adecuados hubo mejora inmediata que ha
continuado hasta la fecha actual.

Hiperparatiroidismo primario

Ma. Esther Gutirrez Daz Ceballos,1 Hctor A. Rodrguez Martnez,1 Evelyn M. Torres
Acosta,1 Humberto Cruz Ortiz1
1 Unidad de Patologa y Servicio de Endocrinologa del Hospital General de Mxico y de la
Facultad de Medicina, UNAM

22.- Mujer de 25 aos de edad que presenta en la cara diseminado comedones, ppulas y
pstulas, crecimiento excesivo de vello en mejillas mentn y cuello, acompaada de seborrea,
refiere cada de pelo exagerada. Inici desde la adolescencia. El diagnstico clnico es:

a) Acn e Hirsutismo
b) Sndrome de masculinizacin
c) Lupus eritematoso discoide
d) Alopecia androgentica

El hirsutismo es el crecimiento excesivo de vello terminal en mujeres siguiendo un patrn


masculino de distribucin, en zonas andrgeno-dependientes: patillas, barbilla, cuello, areolas
mamarias, trax, en rea inmediatamente superior o inferior al ombligo, as como en muslos,
espalda. Frecuentemente se asocia a acn, calvicie con patrn masculino (alopecia
andrognica) e irregularidades menstruales.

Es un trastorno que afecta aproximadamente al 10% de las mujeres en edad frtil, y puede ser
leve, lo que representa una variacin del patrn de crecimiento normal, y en raras ocasiones es
signo de un trastorno subyacente grave.

Por lo general, es idioptico, pero puede estar relacionado al exceso de andrgenos, como el
sndrome de ovario poliqustico o la hiperplasia suprarrenal congnita.

Los pacientes con acn presentan de un modo ms o menos frecuente seborrea de cuero
cabelludo, con o sin pitiriasis.

Otras alteraciones andrgeno-dependientes pueden asociarse, pero no de un modo habitual,


como sucede con el hirsutismo y la alopeca, que pueden presentarse en menos del 10% de
las pacientes.

En el Sndrome de Cushing Hipofisario, la ACTH estimula la produccin suprarrenal de


hormonas andro-gnicas, resultando esto en acn e hirsutismo.

En el sndrome de poliquistosis ovrica (Stein-Leventhal), hay un incremento de la secrecin


gonadal de andrgenos a nivel del hilio y de la teca interna. La hipersecrecin de andrgenos
dotados de bioactividad puede inducir acn e hirsutismo. La presencia de acn en una mujer -
adolescente o adulta - con niveles de testosterona srica superiores a 300 ng/dl sugiere la
presencia de un tumor ovrico.

En la Anorexia Nerviosa, el acn se desencadena generalmente en la fase de recuperacin,


donde puede coexistir con un ovario poliqustico. En fase de amenorrea LHRH, LH, FSH,
estrgenos y progesterona estn disminudos. El cortisol plasmtico est elevado con vida
media prolongada y prdida de la variacin diurna, otro factor que puede producir acn.
No hay que olvidar aquellos sndromes donde el acn coexiste con importantes trastornos
osteoarti-culares, como ser el Sndrome de Apert y el Sndrome SAPHO.

El acn se halla generalmente presente en los pacientes epilpticos - tal vez provocado en
gran medida por la medicacin - y en pacientes psiquitricos (manaco-depresivos y obsesivo-
compulsivos). Es importante tener en cuenta la asociacin de una neurosis de angustia
inespecfica y el acn.

1. Azziz R, Carmina E, Sawaya ME. Idiopathic hirsutism. Endocr Rev 2000;21:347-62.


Hirsutismo. Gonzalez Guerra. Servicio de dermatologa.Fundacin Jimnez Daz (Madrid)

23.- Se trata de femenino de 39 aos la cual presenta amenorrea secundaria de 2 aos y


medio de evolucin. Los niveles reportados de prolactina son de 150ng/ml (normal hasta 20
ng/ml). La resonacia magntica detecta macrotumor de 2,8 cm. de dimetro con expansin
lateral izquierda. No alteraciones visuales. El tratamiento de eleccin en ste caso es:

a) Ciruga por tratarse de un macrotumor.


b) Tratamiento mdico con agonistas dopaminrgicos.
c) Somatostatina previa a ciruga.
d) Radioterapia hipofisaria previa a ciruga.

Tratamiento:

Los dopaminrgicos han revolucionado el tratamiento del prolactinoma y virtualmente han


dejado fuera a la ciruga; as independientemente del tamao del adenoma la primera opcin
teraputica es la farmacolgica.1,2 Con los dopaminrgicos se consigue en poco tiempo
restaurar el funcionamiento ovrico y corregir la esterilidad, incluso antes de que se normalice
la concentracin de prolactina; asimismo se consigue reducir el tamao del adenoma.
Los dopaminrgicos actan sobre los receptores localizados en las clulas mamotrpicas de la
hipfisis anterior y suprimen la sntesis y secrecin de prolactina con la consecuente
normalizacin del eje hipotlamogonadotropico hipotlamogonadotropico- ovrico. La accin
dopaminrgica puede ocasionar efectos colaterales indeseables como nusea, hipotensin
arterial, constipacin nasal, mareo y estreimiento, los cuales no necesariamente corresponden
con la dosis utilizada, pero s se relacionan con el tipo de dopaminrgico. Est ampliamente
documentada la superioridad de la farmacoterapia para el tratamiento de los prolactinomas;
adems la ciruga es raramente curativa, incluso en el caso de microadenoma.4,5

Cuadro I. Agentes dopaminrgicos que se usan como tratamiento de la


hiperprolactinemia y el prolactinoma.
Genrico Comercial Dosis (mg)
Bromocriptina Parlodel 2.5-5 diaria
Lisurida Dopergin 0.2 diaria
Quinagolida Norprolac 25-50 diaria
Cabergolina Dostinex 0.5 c/4 das

Los adenomas hipofisarios representan el 10% de todos los tumores intracraneales


diagnosticados y 25% de los tumores cerebrales que son intervenidos quirrgicamente. Los
objetivos del tratamiento de un paciente con un adenoma de la pituitaria son: eliminar el efecto
de la masa tumoral (compresin sobre estructuras vecinas) disminuir la produccin excesiva de
hormonas, restaurar la funcin normal de la pituitaria y evitar la recurrencia.
El tratamiento de eleccin para todos los prolactinomas es con un agonista de la dopamina. La
bromocriptina y la cabergolina son efectivas para reducir el tamao del tumor y para restaurar
la funcin gonadal. El tratamiento quirrgico debe recomendarse slo cuando falla el
tratamiento mdico.
Los tumores de la pituitaria productores de hormona de crecimiento son tratados
preferentemente mediante adenomectoma transesfenoidal, pero la normalizacin de los
niveles de HC y de IGF-1 ocurre en menos de la mitad de los pacientes con macroadenomas;
por lo tanto, un importante nmero de pacientes acromeglicos requiere un tratamiento
adicional. Los anlogos de la somatostatina son en la actualidad los medicamentos que ms
usados para el control de la acromegalia. En grupos especiales de pacientes, el tratamiento
con agonistas de la dopamina y somatostatina parece que suprimen mejor los niveles de HC
que cuando se administran esos frmacos en forma separada

Figura 1. Paciente de 20 aos quien consult por amenorrea primaria. El estudio de RMN
muestra un macroprolactinoma que invade el seno cavernoso izquierdo y envuelve la
cartida del mismo lado.

RMN de control 10 meses despus de tratamiento con un agonista dopaminrgico. Corte


coronal en T1. No se observa tumor. Tallo hipofisario central y quiasma ptico libre.

REFERENCIAS BIBLIOGRFICAS

Gac Md Mx Vol. 140 No. 5, 2004


Referencias
1. Schlechte JA. Prolactinoma. N Engl J Med 2003;349:2035-2041.
2. Zrate A, Canales ES, Jacobs LS, Soria J, Daughaday WH. Restoration of ovarian
function in patients with the amenorrhea-galactorrhea syndrome after long-term therapy with L-
Dopa. Fertil Steril 1973;24:340.
3. Tyson JE, Carter JN, Andreassen B, Huth J, Smith B. Nursing mediated

24.- Un hombre de 55 aos presenta dolor precordial que le apareci cuando estaban en
reposo; se irradio al cuello y al hombro izquierdo, refiere que tuvo 20 minutos de duracin y
cedi, tiene antecedentes de obesidad, Diabetes Mellitus, hipertensin y sedentarismo, el
diagnstico ms probable es:

a) angina inestable.
b) infarto agudo al miocardio.
c) angina estable
d) pericarditis.

Entendemos por angina de pecho, o angor, un dolor torcico, generalmente retrosternal, que
puede ser descrito como opresivo, quemazn o simplemente una leve pesadez, y que est
motivado por la isquemia del miocardio. Este dolor o molestia puede irradiar o presentarse
nicamente en el cuello, mandbula, hombros, brazos, antebrazos, manos, espalda o
epigastrio. Rara vez se presenta por encima de la mandbula o debajo del epigastrio. La
isquemia miocrdica se presenta en ocasiones con sntomas distintos de la angina como
disnea, debilidad, fatiga o eructos. Estos sntomas son equivalentes anginosos y suelen ser
ms frecuentes en personas ancianas.

La situacin en que se presenta la angina inestable no parece relacionada con un mayor


trabajo cardaco. Es decir, la isquemia miocrdica no parece justificarse por un mayor consumo
miocrdico de oxgeno y, por lo tanto, la causa es una disminucin aguda del flujo sanguneo
coronario. Clsicamente se denomina angina inestable a la que se presenta en las siguientes
circunstancias:

1. Angina de reposo: ocurre en reposo o con un mnimo esfuerzo.


2. Angina de comienzo reciente: inicio de los sntomas en el ltimo mes, en un paciente
previamente asintomtico, y de aparicin con esfuerzos mnimos.
3. Angina progresiva: En un paciente con angina estable previa, los sntomas se
presentan con esfuerzos menores, son cada vez ms frecuentes o su duracin es ms
prolongada.

La presentacin clnica de la angina inestable puede ser idntica a la de un infarto agudo de


miocardio (IAM), la diferencia es conceptual: si hay necrosis miocrdica hablamos de infarto
agudo de miocardio. Los datos que podemos obtener de la historia clnica, la exploracin fsica
y el electrocardiograma, con frecuencia no permiten diferenciar entre angina inestable e infarto
agudo de miocardio. La elevacin de marcadores de dao miocrdico como la
creatnfosfoquinasa (CPK) o la troponina T o I, identifican una necrosis miocrdica y, por lo
tanto un IAM. Es por este motivo que actualmente tanto la angina inestable como el IAM se
agrupan bajo el trmino de sndrome coronario agudo. Segn la presentacin
electrocardiogrfica, el sndrome coronario agudo (SCA) se divide en SCA con elevacin del
segmento ST (frecuentemente evoluciona a un infarto con onda Q) y SCA sin elevacin del
segmento ST, que incluye a la angina inestable y la mayor parte de los casos de IAM sin onda
Q. Esta terminologa es la que actualmente se utiliza porque tiene la ventaja de clasificar el
cuadro clnico del paciente a partir de datos clnicos y electrocardiogrficos que pueden
obtenerse de modo rpido y sencillo.

LECTURA RECOMENDADA
Guas clnicas para el manejo de la angina inestable e infartosin elevacin del ST.
Estratificacin del riesgo
Bibliografa Internacional
R. Marrn Tundidor*, P. Palazn Saura*, L. M. Claraco Vega*, C. Ascaso Martorell*, J.
Povar Marco*,
J. M. Franco Sorolla*, I. Calvo Cebollero**
*SERVICIO DE URGENCIAS Y **SERVICIO DE CARDIOLOGA-UNIDAD DE
HEMODINMICA Y CARDIOLOGA INTERVENCIONISTA.
25.- Se trata de masculino de 60 aos que inicia con dolor precordial sbito irradiado a brazo
izquierdo, cuello y madbula, sudoracin profusa y ansiedad, con antecedentes de tabaquismo
postivo de 20 aos de evolucin consumiendo 1 cajetilla al da, obesidad grado I, niega otras
patologas. Se sospecha de IAM por lo que se procede a toma de ECG, el siguiente trazo
electrocardiogrfco es compatible con:

a) IAM Postero-lateral
b) Angina inestable
c) IAM lateral Alto
d) IAM cara anterior

Electrocardiograma (EKG) de 12 derivaciones de un paciente con un infarto agudo del


miocardio de cara anterior. Ntese el supradesnivel convexo del segmento ST desde V1- V5,
con elevacin del punto J que alcanza los 6 mm en V3.

Diagnstico topogrfico del infarto agudo de miocardio (IMA): 5, 6

Infarto anterior extenso: DI, aVL, V1, V2, V3, V4, V5, V6
Infarto anteroseptal: VI, V2, V3, V4
Infarto anterolateral: DI, aVL, V4, V5, V6
Lateral alto: DI, aVL
Lateral bajo: V5, V6
Infarto inferior: Se observa en DII, DIII, aVF
Infarto posterolateral: DII, DIII, aVF, V5 y V6
26.-Se trata de un hombre de 26 aos, que 10 das despus de acudir a una despedida de
soltero, comienza con inflamacin de rodilla derecha y de ambos tobillos, conjuntivitis bilateral,
aftas orales y erosiones superficiales no dolorosas en el glande.
El diagnstico ms probable es:

a) Enfermedad de Still.
b) Enfermedad de Reiter
c) Infeccin gonoccica.
d) Infeccin por Staphylococcus Aureus.

Proceso inflamatorio estril de la membrana sinovial, precedido o precipitado por una infeccin
que ocurre fuera de la articulacin

DIAGNOSTICO:
HISTORIA
SNTOMAS GENERALES
MANIFESTACIONES MSCULO-ESQUELTICAS
Artralgias, artritis aditiva o migratoria
Monoartritis u oligoartritis asimtrica
Articulaciones grandes que sostienen peso: rodillas, tobillos y caderas
Dactilitis o dedos en salchicha
Afeccin axial: articulaciones S-I y columna lumbar
Entesopata, tenosinovitis

MANIFESTACIONES GENITO-URINARIAS Uretritis, balanitis circinada
(es importante sealar que las lesiones son indoloras) , prostatitis
Cervicitis, cistitis, enfermedad plvica inflamatoria

MANIFESTACIONES CUTNEAS Y DE MEMBRANAS MUCOSAS


Queratodermia blenorrgica
Eritema nodoso
Distrofia ungueal
lceras orales


MANIFESTACIONES OCULARES
Conjuntivitis y uvetis.
MANIFESTACIONES G-I

BIBLIOGRAFA RECOMENDADA

Klippel JH, Stone JH, Crofford LJ, White PH, editors. Primer on the rheumatic diseases.
13th ed. New York: Springer-The Arthritis Foundation; 2008.

Martnez-Elizondo P, editor. Introduccin a la Reumatologa. 4a ed. Mxico: Colegio Mexicano


de Reumatologa A.C./Intersistemas S.A. de C.V.; 2008.

Firestein GS, Budd RC, Harris ED Jr, McInnes IB, Ruddy S, Sergent JS, editors. Kelleys
Textbook of Rheumatology. 8th ed. Philadelphia: Saunders Elsevier; 2009.
27.- Se trata de masculino de 25 aos, con antecedentes de episodios recurrentes de dolor
ocular, fotofobia y lagrimeo, que desarrolla dolor insidioso y progresivo en regin lumbar con
exacerbacin nocturna en cama. A la exploracin fsica: test de Schber positivo y soplo de
regurgitacin artica grado II-III/VI. Rx de trax: sugerente de retraccin fibrosa apical derecha.
El diagnstico ms probable es:

a) Sndrome de Reiter
b) Osteoartrosis
c) Enfermedad de Whipple
d) Espondilitis anquilosante

La espondilitis anquilosante es un padecimiento reumtico inflamatorio, generalizado y


crnico, que afecta primordialmente el esqueleto axial, con la presencia de dao de las
articulaciones S-I (sacroiliitis) como su hallazgo fundamental
PREVALENCIA: Africanos y esquimales: 0.1 %. Blancos: 0.5 1 %. Indios Haida
(Norte de Canad): 6 %
PREVALENCIA DEL HLA-B27: 6 8 %
PREVALENCIA EN FAMILIARES DE ENFERMOS, HLA-B27 +: 10 20 %
INCIDENCIA: 6.3 6.9/100 habitantes por ao
CONCORDANCIA en gemelos: Monocigotos, 63 %; dicigotos, 12.5 %
EDAD: Se inicia en la adolescencia o la edad adulta temprana. Rara despus de los
40 aos
GNERO: 3 5:1 a favor del hombre
SACROILIITIS:

Infiltrado de linfocitos T CD4+ y CD8+, as como macrfagos


RNAm de TNF abundante cerca de los infiltrados
RNAm de TGF- cerca de las reas de formacin de hueso nuevo

Tejido de granulacin en mdula sea subcondral


Hallazgos precoces: sinovitis, inflamacin MO subcondral
Hallazgos avanzados: destruccin extensa del cartlago y el hueso subcondral

COLUMNA VERTEBRAL: osteitis, sindesmofitos, acuadramiento


ENTESITIS
ARTICULACIONES PERIFRICAS: hiperplasia sinovial, infiltrado linfoide, fibrosis y
lesiones vasculares

OCULARES: uvetis anterior. En la mayora de los enfermos se produce durante los
primeros 10 aos de evolucin.
En general es unilateral, con tendencia a recidivar. Cursa con dolor, fotofobia y lagrimeo.
No suele dejar secuelas.
CARDIO-VASCULARES: aortitis ascendente, insuficiencia artica, anormalidades de la
conduccin.
La ms caracterstica es la insuficiencia artica por inflamacin de la aorta y de la
vlvula artica. Es ms frecuente en la espondilitis anquilosante de larga duracin,
especialmente en las que cursan con artritis perifrica importante y con manifestaciones
generales (fiebre, adelgazamiento y anemia). Otras manifestaciones son la insuficiencia
cardiaca, la cardiomegalia y los defectos de conduccin.
PULMONARES: fibrosis apical
GASTRO-INTESTINALES: EII
NEUROLGICAS: complicaciones por fracturas y luxaciones, sndrome de la cola de
caballo
RENALES: amiloidosis, nefritis por IgA, uso de AINE, prostatitis inespecfica
OTRAS
Braun J, et al. Arthritis Rheum 1995;38:499-505.

28.- Mujer de 52 aos de edad, originaria de Granada, Espaa y radicada en la Ciudad de


Mxico refiere rubor, tumefaccin y rigidez de las articulaciones interfalngicas dstales desde
hace tres meses, pero no presenta otras molestias articulares. De los siguientes diagnsticos el
ms probable es:

a) Osteoartritis erosiva

b) Artritis reumatoide

c) Espondilitis anquilosante

d) Esclerodermia

Allen R. M. MMS Medicina Interna. 5. Edicin. National Medical Series. Mc. Graw Hill. 2006.
(captulo 10 V E 1). Tpicamente, la osteoartritis erosiva afecta las articulaciones interfalngicas
dstales en mujeres de edad madura. Es improbable que esos sntomas articulares dstales
prominentes sucedan en pacientes con artritis reumatoide o con lupus eritematoso diseminado
sin molestias articulares ms generalizadas. No hay pruebas que indiquen espondilitis
anquilosante o esclerodermia.

29.- Hombre de 58 aos de edad con una historia de fumar 20 cajetillas/ao, est siendo
evaluado para la colocacin de un bypass coronario. De las siguientes pruebas preoperatorias
la mejor para valorar funcin respiratoria en este paciente ES

a) FVC
b) PaCO2
c) PaO2
d) FEV1/FVC

La relacin FEV1/ FVC le proporciona la capacidad de las funciones pulmonares del paciente y
la FEV1 le indica si hay una obstruccin de la va area, la FVC Y VR aisladas le indican si hay
una obstruccin nicamente, la PaCO2 Y PaO2 solo le indican si hay retencin de Co2 y la
oxemia del paciente.

1.- Fishman AP, , Fishman JA, Grippi MA, Kaisser LR, Seor RM. Pulmonary Diseases and
disorder. 3a. Edicin McGraw-Hill, EUA, 2006.
2.- Fraser, R ; Neil, C; Par, P; Diseases of the Chest, Third Edition, Editorial Elsevier, 2005.
3.- Murray and Nadels; Textbook Respiratory Medicine, Vol 1-2, Elsevier editorial, 2005.
30.- Hombre de 64 aos de edad con antecedente de bronquiectasia acude al departamento
de urgencias con hemoptisis. Refiere tos acentuada y produccin de esputo en la ltima
semana acompaados de febrculas. A menudo su esputo se tie de estras de sangre, pero el
ltimo da observ que est tosiendo sangre coagulada en cantidades equivalentes a una
cucharada, con un total aproximado de una taza en 24hrs. La exploracin fsica muestra signos
vitales normales, con saturacin de oxgeno de 98% en aire ambiente. El paciente manifiesta
disnea leve y sibilancias exhalatorias difusas. La radiografa de trax, adems de mostrar
bronquiectasias, es normal.
El tratamiento inmediato ms apropiado para esta hemoptisis es:

a) Embolizacin de la arteria bronquial


b) TAC de trax
c) Broncoscopia
d) Reseccin quirrgica

En este caso se debe realizarle una broncoscopia con la finalidad de localizar el sitio de
sangrado y detener el mismo por compresin con un cateter de fogarty, la TAC no tiene
indicacin como tratamiento.

TABLA 1. Indicaciones de la broncoscopia diagnstica


Sntomas clnicos no explicados
Tos persistente
Disnea / sibilantes no justificados
Hemoptisis
Disfona
Asma unilateral, de comienzo sbito
Sndrome de vena cava
Parlisis diafragmtica
Aclaracin de una sospecha diagnstica
Alteraciones radiolgicas
Investigar el origen de una citologa de esputo positiva
Estadiaje del carcinoma broncognico
Sospecha de fstula traqueoesofgica
Evaluacin de la va area tras un traumatismo torcico
Evaluacin de la va area en quemados
Toma de muestras en una enfermedad intersticial
Estudio microbiolgico de infecciones respiratorias
Evaluacin de la respuesta al tratamiento del cncer de pulmn
BRONCOSCOPIA DIAGNSTICA Y TERAPUTICA
Monografas NEUMOMADRID
Prudencio Daz-Agero lvarez
Javier Flandes Aldeyturriaga
VOLUMEN X / 2007

31.- Paciente femenino de 29 aos de edad, que acude a consulta por presentar dolor en el
recto, sin encontrarse una causa orgnica despus de mltiples exploraciones mdicas.
Adems refiere que desde hace 4 aos ha presentado vmitos, dolor abdominal generalizado,
palpitaciones, mareos, disfagia, visin borrosa, dolor en los miembros inferiores, dismenorrea y
dispareunia. Se observa triste y ansiosa. El diagnstico ms probable es:

a) Trastorno de ansiedad crnico


b) Depresin crnica con somatizacin
d) Trastorno de conversin
c) Trastorno de somatizacin
Lpez- Ibor J J, Valds M M. Manual diagnstico y estadstico de los trastornos mentales
(DSM- IV). Masson 2005. 545 574. El Trastorno de somatizacin se caracteriza por la
presencia de muchos sntomas somticos que no pueden explicarse por los hallazgos fsicos o
de laboratorio. Comienza antes de los 30 aos, puede perdurar durante aos, es crnico y va
asociado a malestar psicolgico, a un deterioro del funcionamiento social y laboral y a la
bsqueda excesiva de ayuda mdica. Para hacer el diagnstico deben presentarse dolor en
cuatro zonas del cuerpo, dos sntomas gastrointestinales, un sntoma sexual y un sntoma
pseudoneurolgico. La ansiedad y la depresin son las patologas psiquitricas ms
prevalentes.

32.- Mujer de 30 aos que refiere datos de ansiedad, enojo e inseguridad refiriendo que existe
una constante sensacin de que en su trabajo no realiza correctamente las funciones que se
encomiendan. Esto hace que necesite permanentemente revisarlas una y otra vez, lo que le
supone prdida de tiempo y eficacia. Esta sensacin es tan dominante en su psiquismo que le
conduce a la idea de fracaso y a la prdida de autoestima. El trastorno inicial de sta paciente
es:

a) Psictico paranoide
b) Por ansiedad fbica
c) De personalidad evitativo-fbica
d) Obsesivo-compulsivo

CRITERIOS DEL DSM-IV PARA EL DIAGNSTICO DEL TRASTORNO OBSESIVO-


COMPULSIVO. Fuente AMERICAN PSYCHIATRIC ASSOCIATION

Criterios para el diagnstico de F42.8 Trastorno obsesivo-compulsivo (300.3)

A. Se cumple para las obsesiones y las compulsiones:

Las obsesiones se definen por 1, 2, 3 y 4:

1. pensamientos, impulsos o imgenes recurrentes y persistentes que se experimentan en


algn momento del trastorno como intrusos e inapropiados, y causan ansiedad o malestar
significativos
2. Los pensamientos, impulsos o imgenes no se reducen a simples preocupaciones excesivas
sobre problemas de la vida real
3. La persona intenta ignorar o suprimir estos pensamientos, impulsos o imgenes, o bien
intenta neutralizarlos mediante otros pensamientos o actos
4. La persona reconoce que estos pensamientos, impulsos o imgenes obsesivos son el
producto de su mente (y no vienen impuestos como en la insercin del pensamiento)

Las compulsiones se definen por 1 y 2:

1. comportamientos (p. ej., lavado de manos, puesta en orden de objetos, comprobaciones) o


actos mentales (p. ej., rezar, contar o repetir palabras en silencio) de carcter repetitivo, que el
individuo se ve obligado a realizar en respuesta a una obsesin o con arreglo a ciertas reglas
que debe seguir estrictamente.

2. El objetivo de estos comportamientos u operaciones mentales es la prevencin o reduccin


del malestar o la prevencin de algn acontecimiento o situacin negativos; sin embargo, estos
comportamientos u operaciones mentales o bien no estn conectados de forma realista con
aquello que pretenden neutralizar o prevenir o bien resultan claramente excesivos

B. En algn momento del curso del trastorno la persona ha reconocido que estas obsesiones o
compulsiones resultan excesivas o irracionales.

Nota: Este punto no es aplicable en los nios.

C. Las obsesiones o compulsiones provocan un malestar clnico significativo, representan una


prdida de tiempo (suponen ms de 1 hora al da) o interfieren marcadamente con la rutina
diaria del individuo, sus relaciones laborales (o acadmicas) o su vida social.

D. Si hay otro trastorno, el contenido de las obsesiones o compulsiones no se limita a l (p. ej.,
preocupaciones por la comida en un trastorno alimentario, arranque de cabellos en la
tricotilomana, inquietud por la propia apariencia en el trastorno dismrfico corporal,
preocupacin por las drogas en un trastorno por consumo de sustancias, preocupacin por
estar padeciendo una grave enfermedad en la hipocondra, preocupacin por las necesidades
o fantasas sexuales en una parafilia o sentimientos repetitivos de culpabilidad en el trastorno
depresivo mayor).

E. El trastorno no se debe a los efectos fisiolgicos directos de una sustancia (p. ej., drogas,
frmacos) o de una enfermedad mdica.

Especificar si:

Con poca conciencia de enfermedad: si, durante la mayor parte del tiempo del episodio
actual, el individuo no reconoce que las obsesiones o compulsiones son excesivas o
irracionales.

33.-Masculino de 39 aos, acude a consulta por presentar desde hace tres das tos productiva,
dolor torcico, disnea y fiebre. Sin antecedentes patolgicos de importancia.
A la exploracin fsica matidez sobre el hemotrax derecho y estertores gruesos en la misma
rea. Signos vitales: TA 120/80 mmHg, FC 115, FR 28, Temp. 39.1 C. Radiografa de trax
con imagen compatible a consolidacin en lbulo superior derecho.
El siguiente paso en el manejo de este paciente es:

a) Azitromicina va oral.

b) Cultivo de esputo.

c) Penicilina G benzatnica IM.

d) Hemocultivo.
MACROLIDOS
a.- Claritromicina 500 mg VO / 12 hrs
b.- azitromicina 500 mg VO / 24 hrs
c.- Telitromicina 800 mg VO / 24 hrs

BETA LACTAMICOS
a.- Amoxicilina-Ac Clavulanico 500 mg VO / 8 hrs
AMBULATORIO 875 mg VO cada 12 hrs

QUINOLONAS
a.- Levofloxacino 500 mg VO / 24 hrs
b.- Moxifloxacino 400 mg VO / 24 hrs

NEUMONIA ADQUIRIDA EN LA COMUNIDAD:

A) Paciente en tratamiento ambulatorio


a. Paciente sin comorbilidades, sin uso crnico de antibiticos en los tres meses previos
I. Primera opcin: macrlido. Si existe evidencia de encontrarse en una zona con alto ndice (>
25%) de Streptococcus pneumoniae resistente a los macrlidos, se podr utilizar una
fluoroquinolona respiratoria (moxifloxacina, gemifloxacina)
II. Segunda opcin: doxiciclina
b. Paciente con alguna de las siguientes comorbilidades: enfermedad crnica cardiaca,
pulmonar, heptica y/o renal; inmunodepresin por enfermedad o por uso de medicamentos;
uso de antimicrobianos en los tres meses previos
I. Primera opcin: fluoroquinolona respiratoria (moxifloxacina, gemifloxacina)
II. Primera opcin: un lactmico (cefotaxima, ceftriaxona, o ampicilina/sulbactam) en
combinacin con un macrlido.
Armitage K, Woodhead M. New guidelines for the management of adult community-
acquired pneumonia. Curr Opin Infect Dis. 2007;20:170-176.

34.- Se siguieron por un periodo de 10 aos a 500 hombre hipertensos y 500 hombres sin
hipertensin para detectar evento vascular cerebral (EVC). Durante el estudio 80 de los
hipertensos tuvieron EVC (Incidencia de 160 por 1000) y 30 sin hipertensin presentaron el
evento (Incidencia 60 por 1000), el RR fue de 2.66. Este es un ejemplo de un estudio:

a) Transversal
b) Ensayo clnico controlado
c) De casos y controles
d) Cohorte

En los estudios de cohorte se eligen dos grupos uno de expuesto y otro de no expuestos los
cuales son seguidos a travs del tiempo para detectar las posibles consecuencias.
Posteriormente se analiza la informacin calculado la incidencia en el grupo de expuestos y en
el grupo de no expuestos; y una vez obtenidos se calcula el Riesgo Relativo.
Ruiz M. A. Epidemiologa Clnica, Panamericana, 1. Ed. 2004; pgs: 287-289
35.- En un estudio de cohorte, los sujetos a estudiar deben cumplir con la siguiente caracterstica:

a) Tener una historia familiar de la enfermedad en estudio


b) No tener la enfermedad en estudio
c) Que sean adultos con mltiples enfermedades a estudiar
d) Pertenecer a un grupo control

El estudio de cohorte es el diseo ideal para buscar la causa de una enfermedad por lo que
compara a un grupo de sujetos, libres de la enfermedad en estudio expuestos a los factores de
sospecha con otro grupo de sujetos tambin libres de la enfermedad, que no estn expuestos a
los factores sospechosos.
Ruiz M. A. Epidemiologa Clnica, 1. Ed. 2004; pg: 279,280

36.- Los Cipreses es una comunidad de 100,000 personas. Durante 2008 hubo 1,000
defunciones por todas las causas. Durante el mismo ao se registraron un total de 300 casos
de Infartos Agudos al Miocardio y 60 defunciones por esta misma causa.
La tasa de letalidad por Infarto Agudo al Miocardio es de:

a) 6%
b) 20%
c) 2%
d) 60%

La letalidad es una proporcin que indica la importancia de la enfermedad en trminos de su


capacidad para producir la muerte, se expresa como tasa y se reporta como el porcentaje de
muertes de una causa especfica con respecto al total de enfermos de esa causa. (TL= 60 / 300
x 100)

Moreno A. Principales medidas en epidemiologa. Rev Salud Pblica Mex, 2000;42(4): 342.

37.- Is the most common genetic feature in ankylosing spoondylitis

a) BRCA-1
b) HLA B27
c) HNF 4alfa
d) K-ras de novo mutation

La espondilitis anquilosante es un padecimiento reumtico inflamatorio, generalizado y


crnico, que afecta primordialmente el esqueleto axial, con la presencia de dao de las
articulaciones S-I (sacroiliitis) como su hallazgo fundamental

Races griegas: ankilos (inclinado-fusin), espondilos (vrtebra)


PREVALENCIA: Africanos y esquimales: 0.1 %. Blancos: 0.5 1 %. Indios Haida
(Norte de Canad): 6 %
PREVALENCIA DEL HLA-B27: 6 8 %
PREVALENCIA EN FAMILIARES DE ENFERMOS, HLA-B27 +: 10 20 %
INCIDENCIA: 6.3 6.9/100 habitantes por ao
CONCORDANCIA en gemelos: Monocigotos, 63 %; dicigotos, 12.5 %
EDAD: Se inicia en la adolescencia o la edad adulta temprana. Rara despus de los
40 aos
GNERO: 3 5:1 a favor del hombre

FACTORES GENTICOS:

Antgenos del CPH:

- HLA-B27
- HLA-B60
- HLA-DRB1
- TNF
- Otros de clase III: MICA, TAP, LMP2, LMP7, HSP70, complotipos

Antgenos fuera del CPH:

- Regiones en los cromosomas 1, 2, 6, 9, 10, 16 y 19; 16q

FACTORES AMBIENTALES
ALTERACIONES INMUNOLGICAS

Brown MA, Crane A, Wordswoth BP.


Curr Opin Rheumatol 2003;14:354-60.

38.- 2 year old female, is taken by her mother to the emergency room complaining of
intermittent abdominal pain. Medical history of 6 to 24 bloody evacuations in the last 24 hours.
Vital signs: BP 110/65, CR 90 pm, RR 28 pm, tempetature 36.5 C. The child is irritable, crying
loud and in the physical exploration there is a abdominal mass in the right lower quadrant.
Whath would be the most probable diagnosis?

a) Meckels diverticulum.
b) Apendicular abscess.
c) Acute appendicitis.
d) Intestinal intussusception.

Invaginacin intestinal: Ocurre cuando una porcin del tracto alimentario se pliega dentro del
segmento adyacente. Es la causa ms frecuente de obstruccin intestinal entre los 3 meses y
los 6 aos de edad. El cociente varn:mujer es 4:1. Se produce de forma sbita con dolor
clico paroxstico intenso recurrente a intervalos frecuentes, el 60% de los lactantes expulsan
heces que contienen sangre roja y moco, las deposiciones en confitura de grosella.
Nelson, Tratado de Pediatra, 17 Edicin, Ed. Elsevier, Pg.1242-1243
39.- Femenino de 26 aos consulta por un episodio de hemiparesia izquierda sugerente de
ictus. Entre sus antecedentes refiere un hbito tabquico, no se ha documentado hipertensin
ni hiperglucemia, ha tenido un episodio previo de amaurosis fugax y dos episodios de
tromboflebitis en extremidades inferiores. No refiere antecedentes quirrgicos ni ingesta de
medicacin. Ha tenido tres abortos espontneos. El hemograma y el estudio de coagulacin
son normales. La funcin heptica y renal y los electrlitos son normales. El colesterol total es
de 260 mg/dl (normal <240) y los triglicridos de 160 mg/dl (normal <150). La TC en la fase
aguda no aporta datos significativos. El diagnstico ms probable y conducta a seguir ES:

a) El cuadro corresponde a sndrome antifosfolpido. Solicitara una determinacin de


anticuerpos anticardiolipina
b) Se trata de una endocarditis infecciosa a partir de una tromboflebitis sptica. Iniciara
tratamiento antibitico emprico en espera de los cultivos
c) El cuadro corresponde a un accidente aterotrombtico en una paciente con una
hiperlipemia familiar. El origen ms probable es la cartida. Solicitara un estudio de
troncos supraarticos. Iniciara tratamiento hipolipemiante.
d) Dada la edad, se trata de una enfermedad desmielinizante. Solicitara una resonancia
magntica cerebral

El Sndrome Antifosfolipdico es una entidad adquirida y multisistmica, caracterizada por


hipercoagulacin, en la que las manifestaciones cutneas, en un alto porcentaje, permiten
sospechar su diagnstico; stas se caracterizan por livedo reticularis y los diferentes grados de
necrosis cutnea dependiendo de los vasos afectados, adems de otros hallazgos importantes
como son la trombocitopenia y los antecedentes de abortos. Los marcadores serolgicos son
los anticuerpos antifosfolipdicos que corresponden al anticoagulante lpico y a la
anticardiolipina

SNDROME ANTIFOSFOLIPDICO
CRITERIO MAYOR
- Trombosis arterial
- Trombosis venosa
- Aborto recurrente-muerte intrauterina
- Trombocitopenia
El diagnstico se establece con cualquiera de los criterios mayores y el hallazgo serolgico de
anticoagulante lpico o ttulos altos de anticuerpos anticardiolipina(1,4,9). Peridicamente se
realizan reuniones de consenso para unificar criterios de diagnstico y tratamiento en el SAF.
El SAF puede clasificarse como primario o idioptico cuando no se demuestra una causa
subyacente y secundario a mltiples causas, de las cuales la ms importante es el LES en el
grupo de enfermedades inmunolgicas

1. Gibson GE, Su D, Pittelkow MR. Antiphospholipid syndrome and the skin. J Am Acad
Dermatol. 1997; 36:970-82
2. Griffiths MH, Papadaki L, Neild GH. The renal pathology of primary antiphospholipid
syndrome: a distinctive form of endothelial injury. QJ Med. 2000; 91:457-67
3. Quintero del Ro AI. Antiphospholipid antibodies in pediatrics. Current Reumatol Rep. 2002;
4(5):387-91
4. Nahass GT. Antiphospholipid antibodies and the antiphospholipid antibody Syndrome. J Am
Acad Dermatol. 1997; 36:149-68
5. Piette WW. Antiphospholipid syndrome: the problems and the promise. Br J Dermatol. 2000:
142:1079-83
6. Gezer S. Antiphopholipid syndrome. Dis Mon. 2003; 49(12):696-741
7. Nash MJ, Camilleri RS, Kunka S, McKie IJ, Machin SJ, Cohen H. The anticardio- lipin assay
is required for sensitive screening for antiphopholipid antibodies. J Thromb Haemost. 2004;
2(7):1071-3
40.- Se trata de femenino de 38 aos, que acude a intercosulta al servicio de medicina interna
por presentar obesidad troncular de reciente diagnstico, amenorrea y depresin. En la
exploracin fsica se aprecia facies redondeadas con hirsutismo moderado, TA de 160/100
mmHg y edemas en miembros inferiores. Se realizan examenes de laboratorio los cuales
arrojan los siuentes resultados: tirotropina (TSH) 0,7 mU/ml (N: 0,4-5,0), T4 libre 16.8 pmol/l (N:
9,0-23,0), cortisol libre en orina 11 mg/24h (N: 20-100), cortisol plasmtico 3 mg/dl (N: 5-25) y
corticotropina (ACTH) 9 pmol/l (N: inferior a 52). De las siguientes situaciones la que estar
dando lugar a este cuadro es::

a) Sndrome de Cushing ACTH dependiente.


b) Administracin exgena de glucocorticoides.
c) Hipotiroidismo subclnico.
d) Enfermedad de Cushing.

Los corticosteroides (del lat. cortex, cis, corteza, y esteroide) o corticoides son una
variedad de hormonas del grupo de los esteroides (producida por la corteza de las glndulas
suprarrenales) y sus derivados.

Los corticosteroides estn implicados en una variedad de mecanismos fisiolgicos, incluyendo


aquellos que regulan la inflamacin, el sistema inmunitario, el metabolismo de hidratos de
carbono, el catabolismo de protenas, los niveles electrolticos en plasma y, por ltimo, los que
caracterizan la respuesta frente al estrs.

Estas sustancias pueden sintetizarse artificialmente y tienen aplicaciones teraputicas,


utilizndose principalmente debido a sus propiedades antiinflamatorias e inmunosupresoras y a
sus efectos sobre el metabolismo.

Los efectos metablicos del exceso de glucocorticoides son los siguientes:

1. Aumento de la neoglucognesis y resistencia a la insulina; esto puede


llevar a la diabetes mellitus
2. Aumento del catabolismo proteico; esto puede llevar a la emaciacin,
osteoporosis y adelgazamiento de la piel.
3. Aumento y redistribucin de la grasa corporal: se produce una
obesidad de predominio central, facie de luna, tungo o acmulo dorsal
de grasa, manteniendo extremidades relativamente delgadas.
4. Involucin del tejido linftico y disminucin de la respuesta inflamatoria:
se produce una disminucin de la inmunidad celular y humoral con lo
que aumenta la susceptibilidad a infecciones.
5. Aumento de la secrecin de cido por el estmago lo que lleva a una
predisposicin de lcera gastroduodenal.
6. Retencin de sodio y redistribucin de los fluidos corporales lo que
produce edema e hipertensin arterial.
7. Funcin gonadal: los glucocorticoides afectan la secrecin de
gonadotrofinas. En los hombres disminuye la concentracin de
testosterona. En las mujeres, suprime la respuesta de LH al GnRH, lo
que lleva a una supresin de la secrecin de estrgenos y progestinas,
con anovulacin y amenorrea.

Todos los efectos anteriormente enunciados pueden ocurrir independientes del origen de los
glucocorticoides. La causa ms comn de sndrome de Cushing se debe a la administracin
exgena de dosis farmacolgicas de ellos con fines generalmente antiinflamatorios e
inmunosupresores.
41.- Se tata de paciente femenino de 29 aos. Acude a consulta por presentar cefalea,
cansancio e irregularidades menstruales con ritmo de 36 a 50 x 2-3 das. No se ha podido
embarazar despus de 18 meses de actividad sexual regular. Sin antecedentes importantes.
EF: Campos visuales normales, tiroides aumentada de tamao una vez y aumentada de
consistencia, no presencia de galactorrea. Resto normal. Laboratorio: qumica sangunea, Bh y
electrolitos normales. Prolactina 47 ng/dL (< 25), perfil tiroideo: TSH 18 mUI/ml, T4t: 50 nmol/L
(57.9 a 154.4), T4L: 7.7 pmol/L (9 a 24), T3T: 1.06 nmol/L (1.2 a 2.9), T3L: 1.96 pmol/L (3 a
6.31)
El diagnstico es:

a) Hiperprolactinemia
b) Hipertiroidismo
c) Sndrome de ovarios poliqusticos
d) Hipotiroidismo primario

DIAGNOSTICO

Inicialmente el hipotiroidismo se diagnosticaba mediante la cuantificacin por tcnicas de Radio


Inmuno Anlisis (RIA) de las hormonas circulantes triyodotironina y tiroxina; el proceso era
lento y sometido a muchos factores de error que hacan su sensibilidad y especificidad poco
confiables. Posteriormente, se desarrollaron tcnicas para la medicin de la TSH hipofisiaria
igualmente mediante el RIA lo que mejor en forma importante la sensibilidad para el
diagnstico de esta enfermedad; sin embargo, los niveles de deteccin de la prueba se
encontraban en el orden de 1 IU/ml lo que haca que la prueba no fuera sensible para valores
menores de 1 IU/ml. Debido a esto se crearon tcnicas de segunda generacin mediante la
cuantificacin de TSH por anticuerpos monoclonales y RIA, el IRMA (Immuno Radiometric with
Monoclonal Antibodies) que permiti detectar valores de TSH en rangos de 0.1 IU/ml;
posibilitando desde entonces diagnosticar pacientes con hipertiroidismo primario; pero con la
limitante de que para esta tcnica era imposible detectar valores de TSH menores de 0.1
IU/ml por lo que se cre la medicin de TSH mediante quimioluminiscencia o mtodos
enzimticos, es decir las tcnicas de tercera generacin, las cuales pueden detectar valores de
TSH de 0.01 IU/ml; con lo que se logra el espectro ideal para una prueba de laboratorio que
tiene la capacidad de diagnosticar tanto la hipofuncin como la hiperfuncin(20).

Adems el avance no slo fue en la medicin de TSH sino tambin en las hormonas tiroideas
que han evolucionado simultneamente con la TSH y ya se miden incluso las fracciones libres
de hormonas y las fracciones totales, lo que ha facilitado el manejo de estos pacientes. Gracias
a esta evolucin en tcnicas de laboratorio, el diagnstico de hipotiroidismo primario es
bastante sencillo. Niveles de TSH superiores al valor mximo de la tcnica seran diagnsticos
de la disfuncin; pero no es tan fcil. Cuando tenemos un paciente con toda la sintomatologa
del hipotiroidismo y la TSH se encuentra elevada el diagnstico es obvio; pero podemos tener
pacientes con sntomas muy inespecficos como depresin y con examen fsico normal a
quienes se les encuentran valores de TSH por encima del lmite superior y con hormonas
tiroideas normales. Se trata de un hipotiroidismo o es un valor ligeramente elevado ocasional
de una persona sana (21). Igualmente tenemos otra circunstancia que ha sido descrita con
mayor frecuencia: pacientes con valores de TSH en el lmite superior normal y con dislipidemia
a quienes se les da tratamiento con hormonas tiroideas y su dislipidemia se corrige
manteniendo valores de TSH en rangos normales. Todas las circunstancias anteriores han
hecho que aparezca en el hipotiroidismo primario la expresin de hipotiroidismo subclinico, que
ha sido objeto de reuniones y congresos dedicados exclusivamente a este tema. La sociedad
Europea de Tiroides hace algunas recomendaciones para el manejo de esta situacin que se
consideran tiles como gua (Tabla ).
Tabla. Enfoque del paciente con disfuncin tiroidea de acuerdo a los niveles de hormona
estimulante de la tiroides (TSH).

Si TSH 0.4 Si TSH > 5.0


Si TSH < 0.4m U/L a 2.0 mU/L Si TSH 2.01 a 5.0mU/L mU/L
Normal,
Medir T3 y T4 totales o Repetir Dar tratamiento
libres para diagnstico de cada cinco Medir T4 libre y anticuerpos para
hipertiroidismo. aos antitiroideos hipotiroidismo
1. Si AAT (-) y T4 libre es normal
repetir screening cada ao. Si TSH
es > 4.0mU/l en dos ocasiones dar
tratamiento
2. Si AAT (+) y/o T4 libre esta baja
o normal baja tratar si TSH es
mayor de 3.0 mU/l y observar a los
otros
Tomado de Koutras DA. Subclinical hypothyroidism. En G. Hennemann, E.P. Krenning,
Thyroid International Merck KGaA, Darmstadt 1999 (3), 6-9

42.- Se trata de femenino de 24 aos, se presenta a consulta externa refiriendo aumento


excesivo de peso desde hace 6 meses a la fecha, disminucin de lbido, sensacin de tristeza y
amenorrea de 12 semanas. Exploracin fsica: Obesidad truncal y estras en abdomen. TA
150/100mmHg, FC 85 x, FR 16 x, Temp 37.3C. Laboratoriales: PIE negativo.
El siguiente paso para confirmar el diagnstico de esta paciente es:

a) Prueba de tolerancia a la glucosa.

b) Cortisol plasmtico.

c) Prueba nocturna de supresin a la dexametasona.

d) Potasio srico.

Prueba de supresin con dexametasona:


Este procedimiento se utiliza para establecer la presencia de sndrome de Cushing. La
dexametasona, glucocorticoide potente de manera normal suprime la liberacin hipofisiaria de
ACTH con una cada en los corticosteroides del plasma y orina, y as evala la
retroalimentacin inhibidora del eje HHS. En el sndrome de Cushing este mecanismo es
anormal y la secrecin de esteroides es incapaz de suprimirse en forma normal. La prueba de
supresin noturna con 1 mg de dexametasona, es un estudio adecuado para la deteccin de
sndrome de Cushing.
Se administra 1mg de dexametasona VO, como dosis nica a las 11 pm y se toma una
muestra de plasma en la maana siguiente para determinar el cortisol. Si el valor es > de 10
ug/dl, la causa probables es sndrome de Cushing.
Gardner D, Shoback D, Endocrinologa bsica y clnica de Greenspan, Manual Moderno, 7
Edicin, pgs. 378-381
43.-Se trata de masculino de 46 aos. Acude a la consulta solicitando un examen mdico.
Tiene antecedente familiar de diabetes en madre y dos tas maternas. El paciente es obeso
desde los 20 aos de edad. EF: peso 92 kg, estatura 1.75 m, TA 130/90, FC 78x, obesidad
generalizada. Laboratorio: glucosa 132 mg/dL, resto de qumica sangunea normal. Se solicita
otra glucemia que es de 129 mg/dL y una hemoglobina glucosilada con resultado de 6.9 %.
El diagnstico del paciente es:

a) Diabetes mellitus
b) Intolerancia a la glucosa
c) Tolerancia a la glucosa normal
d) Intolerancia a la glucosa de ayuno

DIAGNSTICO DE DM
1. Sntomas de DM m
ms una glucosa
plasmtica casual 200 mg/dl.
plasm

2. Glucemia de ayuno 126 mg/dl.

3. Hemoglobina glucosilada 6.5%.

4. Glucemia 200mg/dl a las 2 horas


despu
despus de 75 g de glucosa oral.

44.- Se trata de masculino de 62 aos es trasladado por ambulancia al servicio de urgencias,


encontrado en la va pblica una madrugada del reciente invierno, con fuerte aliento alcohlico,
slo responde a estmulos dolorosos, hemipleja braquiocrural derecha. Frecuencia cardiaca
irregular. Se ingresa y se realiza un ECG. Segn los siguientes trazos, el diagnstico
cardiolgico es:
a) IAM cara posterior
b) Fibrilacin ventricular
c) Fibrilacin auricular
d) Angina estable

Electrocardiograma:
- Frecuencia cardiaca 65 lpm.
- AQRS -20, aunque cuesta definirlo categricamente, son ejes llamados indeterminados.
- Ausencia de onda P.
- Lnea de base con oscilaciones irregulares:
- en DII y aVR son a muy alta frecuencia (corresponden a temblor muscular).
- en las otras derivaciones, la frecuencia y configuracin son compatibles con fibrilacin
auricular.
- Los complejos QRS en V4-6 tienen voltaje aumentado, aunque no llegan a cubrir los criterios
de crecimiento ventricular izquierdo.
- En la porcin final del QRS estn encerradas en crculo rojo las ondas J de
Osborn.
- Segmento ST y onda T de difcil definicin en DI-II, aVL, aVF (alteraciones difusas en la
repolarizacin ventricular).
La hipotermia produce enlentecimiento en la despolarizacin ventricular prolongacin en la
fase 2 del potencial de accin-, por lo que se prolonga el periodo refractario; induciendo la
aparicin de las ondas J de Osborn. Las fibras auriculares pueden responder con fibrilacin
cuando son expuestas al fro.
El solo tomar helado y sentir el dolor retroesternal est asociado a inversin en la onda T en
DII-III.
Las ondas J de Osborn o simil- tambin se observan en la hipercalcemia y la angina
vasoespstica.
45.- Masculino de 66 aos, con Insuficiencia Cardiaca por cardiopata hipertensiva, en situacin
estable (en clase funcional I segn grado de disnea), presenta en el Ecocardiograma,
Disfuncin Sistlica (Fraccin de Eyeccin < 35%). Qu grupo de frmacos estara ms
indicado como tratamiento inicial?:

a) Digitalicos.
b) Antagonistas de Calcio.
c) Betabloqueantes.
d) Inhibidores de la ECA.

IC Izquierda:
Disnea al ejercicio, tos, fatiga, ortopnea, DPN, cardiomegalia, estertores, ritmo
de galope, congestin venosa pulmonar.
IC Derecha:
Presin venosa elevada, hepatomegalia, edema, usualmente asociado a IC
Izq.
TIPOS:
Insuficiencia Cardaca Aguda

Edema pulmonar agudo


Shock Cardiognico

Insuficiencia Cardaca Crnica

Estado Fisiopatolgico Caracterizado por la incapacidad del corazn para bombear la


cantidad de sangre necesaria para abastecer el metabolismo celular.
La ICC representa un sndrome clnico complejo caracterizado por alteraciones de funcin
ventricular y de regulacin neurohormonal que se acompaa de:
-intolerancia al esfuerzo.
-Reduccin en la calidad de vida.
-Reduccin de la esperanza de vida.
SINTOMAS:
Disnea
Disminucin capacidad funcional
Sntomas urinarios
Sntomas cerebrales
Sntomas insuficiencia cardaca derecha
PROPUESTADEUNESQUEMADETRATAMIENTO

FibrilacinA. Diurtico

contraindicacinoefecto
IECA adverso

ARA
Siguensntomas BB

Espironolactona12,5a25mg/d Preferibleingresohospitalario
o inicialconseguimiento
mensual
Digoxina0,125mga0,25mg/d

Se ha demostrado claramente la importancia del control neurohumoral en el paciente


con disfuncin ventricular.

Manejo inicial del paciente con ICA


= Diurtico + Vasodilatador.

Solo el paciente con ICA + BGC = Inotrpico.

Nuevos estudios clnicos e investigacin bsica se requiere para buscar nuevas


estrategias de manejo

REFERENCIAS:

Los inhibidores de la enzima conversora de angiotensina Rev Cubana Cardiol Cir Cardiovasc
1997:11;29-47.

46.- Paciente masculino de 73 aos de edad, acude al servicio de urgencias refiere dolor
torcico intenso con irradiacin a cuello y epigstrio de 4 horas de duracin. Se realiza
electrocardiograma el cual muestra trazos de elevacin del segmento ST en I, a VL, V5 y V6.
El tratamiento inicial es:

a) Tromblitico con activador tisular del plasmingeno intravenoso ms heparina.


b) Heparina de bajo peso molecular en dosis teraputicas y aspirina.
c) Tromboltico con activador tisular del plasmingeno intracoronario nicamente.
d) Tromboltico con activador tisular del plasmingeno intravenoso heparina y aspirina.

TRATAMIENTO ESPECFICO
1) Antiagregantes plaquetarios
A. Aspirina:
Administrar cuanto antes, si no se lo hizo en el departamento de emergencias (o sala de
guardia) en dosis de 160-325mg, la primera de ellas masticable o disuelta, y continuar
indefinidamente con aspirina diariamente.
Recomendacin clase l, Evidencia A.
B. Clopidogrel (18):
En caso de estar absolutamente contraindicada la Aspirina (alergia o UGD activa), o tener
manifestaciones gastrointestinales. Dosis: 75mg cada 24 hs precedida de una toma de 4 comp.
de 75mg.
En caso de decidirse reperfusin mecnica se debe administrar dosis de carga de clopidrogel
que ha demostrado su eficacia en prevenir, al igual que la ticlopidina la trombosis aguda de
stents. (evidencia II a).

2) Reperfusin miocrdica
Todo paciente con sospecha razonable de IAM con supradesnivel del segmento ST o BCRI
presumiblemente nuevo, que ingrese dentro de las 12 hs de haber comenzado los sntomas,
debe someterse a reperfusin con fibrinolticos.
(Recomendacin clase 1, evidencia A, Centros A y B) o angioplastia primaria (recomendacin
clase 1, evidencia A, Centros A).
a) Trombolticos
La reperfusin farmacolgica con agentes fibrinolticos es el mtodo standard disponible en la
gran mayora de las instituciones en que se tratan la amplia mayora de pacientes con
sospecha de IAM transmural.
Recomendaciones (1,2).
Clase l:
Paciente con menos de 12hs de evolucin, supradesnivel persistente del ST o BCRI
presumiblemente nuevo
Clase lll:
Contraindicaciones Absolutas:
Trauma reciente, ciruga mayor o trauma ceflico (dentro de las ltimas 2-4 semanas)
Hemorragia gastrointestinal en el ltimo mes
Ulcera pptica activa dentro de los 3 ltimos meses.
Infeccin estreptoccica recientemente demostrada para SK; usar tPA..
Ditesis hemorrgica o enfermedad heptica crnica con hipertensin portal.
Alergia a la estreptoquinasa (conocida).
Tratamiento previo con estreptoquinasa entre los 5 das y 2 aos previos.
Accidente cerebrovascular (ACV) hemorrgico dentro del ao previo.
Embarazo.
Tumor cerebral conocido.
Sospecha de diseccin artica.
Clase llb:
TAS mayor de 180 mmHg.*
TAD mayor de 110 mmHg.*
Puncin arterial no compresible dentro de los 14 das.
Menstruacin activa o lactancia.
Resucitacin cardiopulmonar prolongada (mayor de 10') dentro de las 2- 4 semanas.
Ataque isqumico transitorio en los 6 meses precedentes.
Uso corriente de anticoagulantes en dosis teraputicas (RIN:>2.)
Historia de hipertensin crnica severa.
Si las cifras son menores a 180/110 mmHg luego del tratamiento rpido podrn utilizarse los
agentes trombolticos.

DROGAS:
Estreptoquinasa (SK)
Se administra i.v. 1.500.000 UI en 100cc de Dextrosa 5% entre 30-60
Activador Tisular del Plasmingeno (r-tPA) (Rgimen acelerado) (2a)
Se administra 15 mg en bolo, seguidos de infusin i.v. en dosis de 0,75 mg/kg en los primeros
30 min., no excediendo de 50 mg, y 35 mg en los 60 minutos restantes.
SELECCION DEL FIBRINOLITICO
De acuerdo a las recomendaciones (3a y 4a) se deben categorizar a los pacientes de acuerdo
al riesgo clnico y a la probabilidad de sufrir un ACV Hemorrgico (3b), en enfermos de alto y
bajo riesgo clnico y/o hemorrgico. Esta categorizacin es importante, porque permite tomar
decisiones en determinadas situaciones como lo es el seleccionar uno de los dos trombolticos
(5.6.7.). Lo importante, ms all de puntualizar cul agente es el ms eficaz, es administrarlo, y
hacerlo en el tiempo ptimo.

3. Terapia adyuvante a la reperfusin


Antitrombnicos:
Antitrombnicos Indirectos:
Heparina (1)
Recomendaciones
Clase I
a). Heparina no fraccionada, ajustada por peso, por va i.v. cuando se administra rt-PA como
tromboltico. Se comienza antes de administrar el rt- PA (con objeto de contrarrestar el efecto
protrombtico del agente fibrinoltico) con un bolo de 60 U/kg (mximo 4.000 U) seguido de
infusin i.v. en dosis de 12 U/kg/h con un mximo de 1.000
U/h para pacientes con peso corporal >70 kg,.durante 48hs. Se debe mantener un KPTT entre
50-70'' o 1,5-2 veces el basal. (90'' favorecen el sangrado y no se asocian a beneficio). Para el
seguimiento de la anticoagulacin ver nomograma sugerido. La continuacin de la heparina por
ms de 48hs, estara restringido a pacientes de alto riesgo para tromboembolismo sistmico o
venoso. (El estudio ASSENT lll, en curso, compara los beneficios relativos de heparina no
fraccionada y fraccionada. Los resultados se esperan conocer durante el Congreso Europeo de
Cardiologa en Setiembre del 2001.)
b). Por va subcutnea heparina no fraccionada en dosis de 7.500U dos veces por da
heparina de bajo peso molecular, en todos los pacientes no tratados con trombolticos que
no tengan contraindicacin para heparina.
En los pacientes de alto riesgo para embolia sistmica, la heparina endovenosa es la preferida
para luego continuar con warfarina o acenocumarol va oral.

C. Por va intravenosa en pacientes con alto riesgo de embolismo sistmico tratados con
trombolticos no fibrinoespecficos (SK) . La infusin de heparina debe comenzar cuando el
KPTT retorne a <2 veces del control
(70"), y debe infundirse en dosis necesaria para mantener un KPTT 1,5-2 veces del control
(infusin inicial 1.000U/h).
Luego de 48hs. de administracin se debe considerar el cambio a heparina subcutnea,
warfarina, o aspirina sola.
Clase III (Contraindicaciones)
Heparina endovenosa de rutina dentro de las 6hs a pacientes recibiendo un fibrinoltico no
selectivo
(estreptoquinasa) que no estn en alto riesgo para embolismo sistmico.

4. TRATAMIENTO COADYUVANTE
Nitroglicerina:(1)
No ha sido demostrado en forma convincente un beneficio asociado al uso rutinario de nitratos
en la fase inicial del infarto de miocardio (1.2.8). Su utilizacin no rutinaria est indicada en
infartos transmurales grandes o extensos
(IAM anterior) por 24-48hs (8), isquemia persistente, hipertensin o insuficiencia cardaca. Se
puede continuar ms all de las 48hs. en casos de angina recurrente o congestin pulmonar
persistente (Clase 1).
Debe administrarse con bomba de infusin intravenosa, 10-20 ug/m aumentando la dosis en 5-
10 ug/m cada 5-10', monitorizando cuidadosamente la respuesta clnica y hemodinmica. La
titulacin se puede hacer de acuerdo al control de los sntomas clnicos o hasta la disminucin
de la TAS del 10% en pacientes normotensos o del 30% en aquellos hipertensos. Deben
mantenerse cifras de TAS superior a 100 mmHg y evitarse un aumento de la FC mayor de 10
latidos/m o que exceda los 110 latidos/m. Dosis mayores a 200 g/m deben ser evitados por el
riesgo potencial de inducir hipotensin arterial.

Manejo y Tratamiento del Infarto Agudo de Miocardio con Supradesnivel del Segmento
ST Dr. Ernesto Paolasso, Dr. Vctor Boccanera,
Dr. Marcelo Jimnez K., Dr. Hctor Luciardi,
Dr. Fernando Nol, Dr. Walter Quiroga,
Dr. Hugo Ramos

47.- Femenino de 20 aos de edad presenta trombosis venosa profunda de nueva aparicin en
la extremidad inferior izquierda. Tiene antecedentes de trombocitopenia leve y dos abortos; fue
tratada para sfilis hace dos aos debido a una prueba de reagina rpida del plasma positiva,
aunque el anticuerpo antitreponmico fue negativo. El diagnstico ms probable de sta
paciente es:

a) Sndrome de anticuerpos antifosfolpido


b) Lupus eritematoso diseminado (SLE)
c) Sndrome de anticuerpo Ro
D) Arteritis de Takayasu

Allen R. M. MMS Medicina Interna. 5. Edicin. National Medical Series. Mc. Graw Hill. 2006.
(captulo 10VIIFl,2c, G3b). Esta paciente tiene varias manifestaciones sutiles de sndrome de
anticuerpo antifosfolpido, que en conjunto hacen de ste un diagnstico probable. La prueba
de reagina plasmtica rpida (RPR) positiva, con resultados negativos de la prueba de
treponemas, probablemente refleja anticuerpos que producen reaccin cruzada a la cardiolipina
o a componentes fosfolpidos de antgenos treponmicos. La trombocitopenia es frecuente en
esta situacin, debido a las interacciones de plaquetas y clulas endoteliales y a la coagulacin
inducida por los anticuerpos. Los abortos pueden deberse a coagulacin en vasos placentarios
pequeos. Las trombosis venosas profundas o incluso la coagulacin en arterias mayores
pueden ser causadas por hipercoagulabilidad consecuente. No hay otras pruebas de lupus
eritematoso diseminado, aunque pueden encontrarse anticuerpos antifosfolpido en alrededor
de 33% de los pacientes. Tampoco se describen datos de sndrome de anticuerpo Ro o
enfermedad indiferenciada del tejido conjuntivo. La arteritis de Takayasu se presenta en
mujeres asiticas jvenes, pero no hay datos de isquemia de grandes vasos arteriales
caracterstica de esta enfermedad.

48.-Se trata de paciente femenino de 43 aos de edad con diagnstico de polimiosistis al


realizar la exploracin fsica, la disminucin de la fuerza muscular se caracteriza por:

a) Ser de predominio distal


b) Acompaarse de dolor intenso
c) Ser de predominio proximal
d) Afectar nicamente a la musculatura distal

La polimiositis (PD) y dematomiositis (DM), son enfermedades inflamatorias en las cuales el


compromiso principal es la debilidad muscular, generalmente proximal y simtrica con atrofia
subsecuente, generalmente indolora.

Su etiologa y patogenia siguen siendo desconocidas, siendo considerada dentro de las


enfermedades difusas del tejido conectivo.

Numerosos estudios sugieren que una exposicin previa a diversos virus como Influenzae A y
B, Coxsackie virus tipo B y Picornavirus pueden desencadenar PM-DM; adems se cree que
los fenmenos inmunolgicos juegan un papel preponderante, por la presencia de
autoanticuerpos, depsito de inmunoglobulinas y complemento en las paredes vasculares.

A nivel anatomo-patolgico las principales alteraciones se encuentran en los msculos


esquelticos y en los vasos sanguneos. Siendo la vasculitis un signo comn en la
dermatomiositis infantil y su severidad indicar una peor evolucin. Clnicamente los sntomas
iniciales son la debilidad muscular proximal, el rash tpico, frecuente el edema en cara y a
veces en miembros, fiebre moderada, posteriormente dificultad para la deglucin y debilidad en
la voz; asimismo manifestaciones viscerales que no se presentan siempre, pero son de gran
importancia en cuanto al pronstico, asimismo transtornos ventilatorios (pulmonares): por
afectacin primaria (neumona intersticial) o secundaria a la disfuncin farngea (aspiracin) o a
la debilidad de los msculos respiratorios (insuficiencia ventilatoria), o como complicacin del
tratamiento (infecciones oportunistas).

Wortmann RL. Idiopathic inflammatory myopathies. A. Clinical features. In: Primer on the
rheumatic diseases. New York: Springer-Arthritis Foundation; 2008. p. 363-7.

49.- Masculino universitario de 21 aos de edad acude a consulta debido a dolor torcico,
fiebre, cefalea y dolor muscular durante las ltimas 2 semanas. Refiere que sus compaeros de
casa han desarrollado sntomas similares. Niega el uso de drogas ilcitas y no es homosexual.
Su temperatura es de 38.2C, FC 90lpm, FR 18x. Se auscultan murmullo vesicular bilateral.
Una RX de trax muestra opacidades intersticiales unilaterales. El patgeno ms probable es:

a) Mycoplasma pneumoniae
b) Bacterias anaerbicas
c) Pneumocystis carinii
d) Streptococcus pneumoniae

Neumonia Adquirida en la comunidad :


Etiologia :

1.- Streptococo Pneumoniae 60 %


2.- Haemophilus Influenzae 10 %
3.- Mycoplasma Pneumoniae 6%

4.- Chlamydia Pneumoniae 4%


5.- Staphilococo Aureus 3% u
1.- Fishman AP, , Fishman JA, Grippi MA, Kaisser LR, Seor RM. Pulmonary Diseases and
disorder. 3a. Edicin McGraw-Hill, EUA, 2006.
2.- Fraser, R ; Neil, C; Par, P; Diseases of the Chest, Third Edition, Editorial Elsevier, 2005.
3.- Murray and Nadels; Textbook Respiratory Medicine, Vol 1-2, Elsevier editorial, 2005.

50.- Masculino de 56 aos, indigente, es trado por paramdicos quienes refieren tos purulenta
y mal oliente. A su ingreso al servicio de urgencias saturacin de oxgeno de 72%. En
radiografa de trax se observan opacidades en lbulos medio e inferior de pulmn derecho.
EL tratamiento indicado en este paciente es:
a) Cefuroxime IV.

b) Metronidazol IV.

c) Ceftriaxona IV.

d) Clindamicina IV.

La neumona por aspiracin se desarrolla despus de la inhalacin de material orofarngeo


colonizado. La aspiracin de secreciones colonizadas de la orofaringe es el mecanismo
primario por el cual la bacteria gana entrada a los pulmones.
Los signos de hipoxemia, como la taquipnea, taquicardia y cianosis, surgen en forma inmediata
y persisten durante varias horas. La auscultacin torcica indica la presencia de jadeos,
estertores y roncus, y es factible que el paciente expectore grandes volmenes de esputo
sanguinolento y espumoso. La broncoaspiracin grave da por resultado insuficiencia
respiratoria con una combinacin de acidosis metablica y respiratoria.
Los antibiticos de eleccin para los casos de neumona por aspiracin son la penicilina y la
clindamicina
Georges CB. Neumona por broncoaspiracin, empiema y absceso pulmonar. En: Tintinalli JE,
Krome RL, Ruiz E. Medicina de Urgencias. 4 edicin. Interamericana McGraw Hill. Mxico.

51.- Femenino de 57 aos, refiere vivir sola, como antecedentes refiere HTA y artritis
reumatoide, con buen control farmacolgico. Presenta alucinaciones auditivas y cenestsicas,
con ideas delirantes de perjuicio con los vecinos y de contenido mstico-religioso de 4 meses
de evolucin. El diagnstico ms probable de la paciente es::

a) Esquizofrenia de inicio tardo.


b) Sndrome confusional agudo.
b) Depresin delirante.
d) Psicosis psicgena

LOS LTIMOS AOS se ha incrementado el inters por el estudio de los estados psicticos de
inicio tardo y su relacin con el deterioro cognoscitivo y los procesos demenciales ya
instalados.
Las dificultades en la organizacin de un cuadro clnico especfico de psicosis tarda que
responda a un estado nosolgico, con caractersticas clnicas propias y a una etiologa clara,
que le de cuerpo como una entidad, ha generado confusin entre los diversos investigadores
clnicos y epidemiolgicos.

Esquizofrenia tarda: se caracteriza por la presencia de delirios, con alucinaciones y con


desorganizacin de la personalidad, alteracin afectiva y conductas bizarras. Hay un menor
grado o ausencia de deterioro cognoscitivo, con una edad de inicio entre los 45 y 60 aos. No
hay antecedentes de trastornos psicticos, afectivos o demenciales.
. Esquizofrenia de inicio muy tardo en el anciano: son cuadros clnicos de tipo psictico en
ancianos-ancianos (mayores de 60 aos) que presentan sintomatologa delirante poco
estructurada, con alucinaciones, con ausencia de deterioro cognoscitivo generalizado y
progresivo, sin compromiso afectivo significativo.
Existen otros estados psicticos no esquizofrnicos de aparicin en edades avanzadas, como:
el trastorno delirante de ideas persistentes, la paranoia, el trastorno delirante de perjuicio, los
delirios hipocondracos de enfermedad y los delirios erotomanacos.
Cuadro clnico
Diversos autores han intentado caracterizar un cuadro clnico de las psicosis de inicio tardo en
donde predominan los delirios, las alucinaciones, interpretaciones paranoides (errores de
percepcin) y otros sntomas Scheneiderianos. En ancianos con psicosis tardas el cuadro
clnico puede estar conformado por:

Fenmenos delirantes, especialmente paranoides de tipo persecutorio o de referencia. Pueden


presentar un tipo especial de delirios denominados de tabique, los cuales se caracterizan por la
creencia de que detrs de las paredes de su habitacin operan personas con el propsito de
hacerles dao o conspirar contra sus intereses e interferir contra su vida.
. Fenmeno del eco del pensamiento: manifiestan que sus pensamientos son ledos o robados.
. Presencia de alucinaciones especialmente auditivas, con contenidos de referencia,
persecutorios o erticos. Tambin pueden presentar alucinaciones visuales, tctiles y olfativas.
. Ausencia de trastornos cognoscitivos generalizados y progresivos.
. Los trastornos afectivos no son significativos, de tal modo que se sospeche de cuadros de
trastorno esquizoafectivo; pero estos ancianos psicticos pueden presentar cuadros depresivos
moderados, sin que lleguen a constituirse en una depresin mayor de tipo psictico.
. Edad de inicio por encima de 60 aos.
. Co-morbilidad frecuente con personalidad premrbida esquizoide o paranoide.
. Alteraciones del comportamiento se presentan ms en el perodo de estado de la crisis
psictica, pero en general hay menos desorganizacin y conductas bizarras que en aquellos
con cuadros crnicos de esquizofrenia.
. Falta de insight o introspeccin.
. Signos neurolgicos blandos como reflejo glabelar, temblor, movimientos anormales,
discinesia tarda, hipoacusia, rigidez.
ALARCN R.
Alteraciones Psiquitricas en la Demencias, en Arango LJC, Fernndez GS y Ardila A, Las
Demencias: Aspectos Clnicos, Neuropsicolgicos y Tratamiento, Ed. Manual Moderno, Mxico,
Mxico, 2003
ALMEIDA OP, HOWARD R, LEVY R, DAVID AS. Psychotic states arising in late life (late
paraphrenia). The role of risk factors. Br J Psychiatry,
1995; 166: 215-228
AMORES GF.
Funciones Cognitivas Superiores, Taller Master de Psicogeriatra, Universidad Autnoma de
Barcelona,
Barcelona, Febrero, 2004
CASTLE D, MURRAY RM.
The epidemiology of late onset schizophrenia. Schizophr Bull, 1993; 19: 691-700
CHRISTENSON R, BLAZER DG.
Epidemiology of persecutory ideation in an elderly population in the community. Am J
Psychiatry,
1984; 141: 1088-1091

52.- Se trata de masculino de 5 aos de edad quien es llevado a consulta por sus padres
quienes refieren que el menor lleva varias noches despertndose agitado como si hubiera
soado algo que le angustia. Cuando acuden a su lado por la noche, el nio les mira y dice
palabras que no tienen ningn significado. Al cabo de un rato vuelve a dormirse y por la
maana no recuerda nada de lo ocurrido. El diagnstico ms probable es:

a) Pesadillas
b) Disomnia.
c) Terrores nocturnos.
d) Sonambulismo.

Criterios para el diagnstico de F51.5 Pesadillas (307.47)

A. Despertares repetidos durante el perodo de sueo mayor o en las siestas diurnas,


provocados por sueos extremadamente terrorficos y prolongados que dejan recuerdos
vividos, y cuyo contenido suele centrarse en amenazas para la propia supervivencia, seguridad
o autoestima. Los despertares suelen ocurrir durante la segunda mitad del perodo de sueo.

B. Al despertarse del sueo terrorfico, la persona recupera rpidamente el estado orientado y


despierto (a diferencia de la confusin y desorientacin que caracterizan los terrores nocturnos
y algunas formas de epilepsia).

C. Las pesadillas, o la alteracin del sueo determinada por los continuos despertares,
provocan malestar clnicamente significativo o deterioro social, laboral o de otras reas
importantes de la actividad del individuo.

D. Las pesadillas no aparecen exclusivamente en el transcurso de otro trastorno mental (p. ej.,
delirium, trastorno por estrs postraumtico) y no se deben a los efectos fisiolgicos directos de
una sustancia (p. ej., drogas, frmacos) o de una enfermedad mdica.

Criterios para el diagnstico de F51.4 Terrores nocturnos (307.46)

A. Episodios recurrentes de despertares bruscos, que se producen generalmente durante el


primer tercio del episodio de sueo mayor y que se inician con un grito de angustia.

B. Aparicin durante el episodio de miedo y signos de activacin vegetativa de carcter


intenso, por ejemplo, taquicardia, taquipnea y sudoracin.

C. El individuo muestra una falta relativa de respuesta a los esfuerzos de los dems por
tranquilizarle.

D. Existe amnesia del episodio: el individuo no puede describir recuerdo alguno detallado de lo
acontecido durante la noche.

E. Estos episodios provocan malestar clnicamente significativo o deterioro social, laboral, o de


otras reas importantes de la actividad del individuo.

F. La alteracin no se debe a los efectos fisiolgicos directos de una sustancia (p. ej., drogas,
frmacos) o de una enfermedad mdica.
53.- Masculino de 22 aos, sin antecedentes patolgicos de importancia, inicia un cuadro de
febrcula, dolor articular, tos seca persistente y astenia de dos semanas de evolucin. En el
ltimo mes, sus dos hermanos menores que l han presentado consecutivamente un cuadro
similar, que se ha autolimitado de forma progresiva. Tras practicrsele una radiografa de trax,
el mdico le ha diagnosticado de neumona atpica. Cul es el agente etiolgico ms probable
en este caso?:

a) Mycoplasma pneumoniae.
b) Legionella pneumophila.
c) Coxiella burnetti (fiebre Q).
d) Haemophilus influenzae

La neumona por micoplasma es un tipo de neumona atpica y es causada por la bacteria M.


pneumoniae. Este tipo de neumona generalmente afecta a personas menores de 40 aos.
Diversos estudios sugieren que esta enfermedad comprende entre el 15 y el 50% de todos los
casos de neumonas en adultos e incluso ms en los nios en edad escolar.

Las personas que se encuentran en mayor riesgo de adquirir neumona por micoplasma
incluyen aquellos que viven o trabajan en reas de hacinamiento como escuelas y hogares de
personas abandonadas, aunque muchas personas que la contraen no presentan ningn factor
de riesgo que se pueda identificar.

Cuadro clnico
Mycoplasma pneumoniae produce infecciones del aparato respiratorio, principalmente en forma
de neumona que, por sus peculiares caractersticas de presentacin clnico radiolgica se
denomina neumona atpica primaria. Los sntomas se presentan de manera gradual en varios
das, y consisten en fiebre, tos no productiva, cefalea y mialgias. A menudo, se acompaa de
faringitis, rinitis, otitis y traqueobronquitis. La exploracin fsica se caracteriza por la parquedad
de sntomas, auscultndose ligeros subcrepitantes, aunque los pacientes pueden presentar
crepitantes francos, roncus y sibilantes. En la radiografa de trax se observan infiltrados
retculonodulillares parahiliares o peribronquiales que pueden ser uni o bilaterales. Puede
observarse la presencia de un pequeo derrame pleural en uno de cada cuatro o cinco
pacientes. En los anlisis complementarios suele encontrarse una discreta leucocitosis en un
30% de los pacientes.
Los nios con alteraciones inmunolgicas como la anemia de clulas falciformes, con
anesplenia funcional o con sndrome de Down, pueden desarrollar una infeccin respiratoria
grave y de evolucin fulminante. La hipogammaglobulinemia es tambin un factor de riesgo
para las infecciones del tracto respiratorio y de sus complicaciones a nivel articular.
Se han descrito otro tipo de infecciones, aunque en menor frecuencia. Generalmente,
acompaan a un cuadro respiratorio, pero pueden aparecer en ausencia absoluta de sntomas
de esta localizacin. En la tabla 1 se relacionan las principales manifestaciones y
complicaciones extrapulmonares.

Tabla 1. Infecciones extrapulmonares por Mycoplasma pneumoniae.

Sistema nervioso central


Meningoencefalitis, neuritis ptica, parlisis nervios craneales, parlisis ascendente (Sndrome
Guillain-Barr), ataxia y psicosis
Piel Erupcin eritematosa papular o vesicular. Sndrome de Stevens-Johnson
Articular Mialgias, artralgias y poliartropatias
Artritis sptica (especialmente en caso de hipogammaglobulinemia)
Cardiaca Pericarditis, miocarditis y derrame pericrdico
Sistema hematopoytico
Anemia hemoltica asociada con aglutininas frias
Prpura trombtica trombocitopnica
Renal Glomerulonefritis, nefritis tubulointersticial, nefropata IgA
Gastrointestinal Vmitos, diarreas y hepatitis colestsica. Pancreatitis
Otros Otitis externa, otitis media y miringitis
Rabdomiolisis
Conjuntivitis, uvetis anterior, retinitis y neuritis ptica
Abscesos tubo-ovricos

BIBLIOGRAFA
WAITES KB, TALKINGTON DF. Mycoplasma pneumoniae and its role as a human patogen.
Clin Microbiol Rev 2004; 17:697-728.
AUSINA V, RODRIGO C. Infecciones causadas por micoplasmas. En: Farreras-Rozman (eds).
Medicina Interna, 15 ed. Madrid: Elsevier Espaa SA, 2004; pp 2362-2365.
TALKINGTON DF, SHOTT S, FALLON MT, SCHWARTZ SB, THACKER WL. Analysis of eight
commercial enzyme immunoassay tests for detection of antibodies to Mycoplasma
pneumoniae in human serum. Clin Diagn Lab Immunol 2004; 11:862-867.
MICHELOW IC, OLSEN K, LOZANO J, DUFFY LB, MCCRACKEN GH, HARDY RD. Diagnostic
utility and clinical significance of naso- and oropharyngeal samples used in a PCR assay to
diagnose Mycoplasma pneumoniae infection in children with community-acquired pneumonia. J
Clin Microbiol 2004; 42:3339-3341.
TEMPLETON KE, SCHELTINGA SA, GRAFFELMAN AW, VAN SCHIE JM, CRIELAARD JW,
SILLEKENS
P, ET AL. Comparison and evaluation of real-time PCR, real-time nucleic acid sequence-based
amplification, conventional PCR, and serology for diagnosis of
Mycoplasma pneumoniae. J Clin Microbiol 2003; 41:4366-4371.

54.- En los estudios epidemiolgicos existe un modelo retrospectivo, observacional,


comparativo, que parte del efecto a la causa, que es til cuando se investigan enfermedades
de baja incidencia, que no expone a riesgo a ninguno de los sujetos estudiados y que ofrece un
ndice conocido como riesgo relativo (Odds Ratio). Ese diseo es:

a) La serie de casos
b) Ensayo clnico controlado
c) El reporte epidemiolgico
d) Casos y controles

Crdova VH, Jimnez J, Jimnez MC. Manual de diseo metodolgico en investigacin clnica.
ULSA UAPY 2001. Pg. 30 31. Los estudios de casos y controles son retrospectivos, de
observacin y comparativos. A diferencia de los estudios de Cohorte, parten en esencia desde
el efecto y hacia atrs buscan identificar causas o factores de riesgo o exposicin. Al grupo de
personas que tienen el fenmeno o enfermedad se le denomina casos y se compara con otro
grupo de individuos que no tienen el fenmeno o enfermedad y se les denomina controles
Este tipo de diseo es particularmente til cuando queremos estudiar a pacientes o
poblaciones con alguna enfermedad que se present mucho tiempo despus de haber ocurrido
la exposicin o cuando queremos investigar los factores de riesgo de enfermedades poco
frecuentes o con causas mltiples. En estos estudios se calcula el riesgo relativo (Odds Ratio)
y se interpreta en razn al valor de 1, esto es, menos de uno es poco probable que ocurra el
riesgo relativo y ms de uno es probable que s ocurra.

55.- Al realizar un ensayo clnico controlado doble ciego para evaluar un nuevo antiinflamatorio,
Qu condicin deben cumplir los participantes del estudio?

a) Ni el grupo de estudio ni el grupo control conocen a los observadores


b) Los sujetos del grupo control no conocen a los sujetos del grupo en estudio
c) La asignacin del tratamiento no es conocida por los pacientes
d) Ni el observador ni los sujetos conocen cual grupo recibe el nuevo medicamento y
cual el placebo

Para considerar el efecto de placebo y reducir los sesgos debido a las concepciones de los
pacientes y los investigadores el estudio puede conducirse bajo un patrn ciego. En un estudio
doble ciego, la asignacin al tratamiento no es conocida por los pacientes ni por los mdicos.

Greenberg R. S; Epidemiologa mdica, Manual Moderno, 2. Ed. Pg. 116

56. Masculino de 66 aos que cursa con antecedente de infarto agudo de miocardio, el
siguiente grupo de medicamentos son un tratamiento generalmente indicado, por su alta
disminucin de mortalidad:

a) Betabloqueadores.
b) Nitritos.
c) Anticoagulacin oral.
d) Propafenona

Los BB son recomendados en todos los pacientes que han presentado un IAM,
siempre que no tengan contraindicaciones para su uso, y de modo permanente
(indefinidamente): clase de recomendacin I, nivel de evidencia A.

Se ha puesto en evidencia que los BB son infrautilizados en esta indicacin.

Ms de 35.000 pacientes han sido incluidos en estudios postinfarto con BB.

Se ha demostrado una reduccin de la mortalidad total, muerte sbita y reinfarto del


orden del 20-25%.

Clase de Nivel de
Situacin clnica/indicacin
recomendacin evidencia

Todos los pacientes sin


contraindicaciones, indefinidamente

Para mejorar supervivencia I A


Para prevenir reinfarto I A
Prevencin primaria de la muerte sbita I A
Prevencin/tratamiento de arritmias
IIb B
ventriculares tardas

Los BB siguen estando indicados en el tratamiento de la HTA, aunque existen crticas a


su empleo en el primer escaln.
Pese a ello, son sin duda de primera eleccin si el/la paciente presenta angina, cardiopata
isqumica en general, insuficiencia cardaca, taquiarritmias, glaucoma o embarazo.
Los BB son frmacos de primera eleccin en la insuficiencia cardaca con disfuncin
sistlica. Slo el carvedilol, el bisoprolol o el nebivolol son BB que puedan ser empleados a las
dosis disponibles en nuestro medio para dicho tratamiento.

Todos los pacientes sin contraindicaciones deben recibir BB si presentan cardiopata


isqumica y, muy especialmente, si existe angina o han presentado un IAM.

ESC Expert consensus document on b-adrenergic receptor blockers. The Task Force on Beta-
Blockers of the European Society of Cardiology.
Eur Heart J 2004; 25: 13411362

57.- Un paciente con insuficiencia renal crnica debida a hipotensin prolongada grave es
atendido porque presenta dolor retroesternal. Se le indic hemodilisis dos veces por semana
en los ltimos dos aos y en fechas recientes ha experimentado episodios de hipotensin al
inicio del tratamiento. El dolor se localiza sobre el msculo trapecio. Se reduce un poco al
adoptar la posicin de pie y se exacerba con la respiracin profunda. Cul de los siguientes
trastornos es la causa ms probable del dolor retroesternal que sufre este paciente?

a) Arteriopata coronaria
b) Espasmo esofgico difuso
c) Embolias pulmonares
d) Pericarditis

El dolor torcico que sufri este paciente es caracterstico de pericarditis e inflamacin del
pericardio, complicaciones comunes en personas con insuficiencia renal crnica en
hemodilisis. Estos enfermos tambin pueden tener inflamacin de varios recubrimientos
serosos, como peritoneo y pleura; no se conoce el mecanismo de esta complicacin. Aunque
es comn la arteriopata coronaria en pacientes en dilisis, las caractersticas de dolor en este
individuo sugieren que no es el diagnstico. La enfermedad esofgica tambin es comn en
sujetos en dilisis y debe descartarse especficamente como posible causa. La relacin
aparente con la dilisis, as como la frecuencia de los sntomas, van contra el diagnstico de
embolia pulmonar como causa del dolor torcico. Adems, el dolor musculoesqueltico debido
a diversos trastornos se observa en pacientes en dilisis y puede deberse a anomalas en el
metabolismo del calcio y el fsforo, que produce depsitos de calcio en diversos componentes
del sistema musculoesqueltico.

1. Alexander JS. A pericardial effusion of gold paint appearance due to the presence of
cholesterin. Br Med J 1919; 2: 463
2. Brawley RK, Vasko JS, Morrrow AG. Cholesterol pericarditis: consideration of its
pathogenesis and treatment. Am J Med 1966; 41: 235-248.

58.- Se trata de masculino de 58 aos quien presenta sntomas de probable infarto agudo al
miocardio, Cul de los siguientes marcadores tiene mayor sensibilidad para realizar ste
diagnstico?

a) CPK
b) Troponina
c) Fosfatasa Alcalina
d) Deshidrogenada lctica

Es muy conocida la falta de especificidad de la CPK. La elevacin de la actividad de esta


enzima se produce tanto en el infarto de miocardio (IAM) como en afecciones caracterizadas
por un grado variable de necrosis muscular. Por ello en los ltimos aos se han realizado, y se
siguen realizando, considerables esfuerzos para encontrar nuevos marcadores analticos que
sean capaces de diferenciar estos cuadros. Entre ellos, la determinacin de la isoenzima
miocrdica de la CPK de forma cuantificada (CPK-MB-Masa) resulta tambin poco especfica
ya que se eleva tambin en caso de necrosis de msculo estriado; est descrita su elevacin
en traumatismos, rabdomiolisis, convulsiones1, miopatas agudas y crnicas2, insuficiencia
renal en dilisis3 e incluso en el ejercicio intenso4,5; y lo mismo podemos afirmar de la
mioglobina6.

Ms recientemente se han determinado las troponinas T e I en sus isoformas especficas de


msculo cardiaco, que tienen una secuencia diferente de aminocidos a las de msculo
estriado no cardiaco, lo que permite el desarrollo de inmunoensayos especficos. El complejo
de las troponinas T, C, e I est estrechamente unido al filamento de tropomiosina. La T se
encarga de la unin a tropomiosina; la C es iniciadora de la contraccin tras unirse al calcio, y
la I se llama as por ser inhibidora de la contraccin en reposo7. Esta especificidad ha sido
demostrada en varios trabajos que muestran valores normales de troponina-I en procesos que
cursan con necrosis de msculo estriado2,8.

Adems de ser muy especficas, las troponinas son altamente sensibles en el infarto de
miocardio. Mair y col encuentran una sensibilidad del 100% para el diagnstico si se hace la
determinacin de troponina-I pasadas 6 horas del comienzo del dolor9. La cintica de estos
marcadores en pacientes con IAM puede resumirse de esta forma:

1. La CPK total comienza a elevarse a las 4-8 horas del comienzo de los sntomas, y
permanece elevada hasta que se normaliza a las 48-72 horas10.

2. La isoenzima MB de la CPK (CPK-MB) aparece en el suero tres horas despus del comienzo
del IAM, con un pico mximo a las 18-20 horas, alcanzando valores 16 veces superiores al
normal. A partir de este punto desciende lentamente y persiste elevada al menos 2 das11.

3. La troponina-I se eleva a partir de las 2-3 horas del comienzo de los sntomas, con un valor
mximo a las 16 horas. Desciende bruscamente hasta las 48 horas, y a partir de entonces se
produce un lento descenso; puede detectarse todava el 7-8 da11.

4. La mioglobina es la primera que se eleva. Da las cifras ms altas de sensibilidad en el plazo


de dos horas del comienzo del dolor10 con respecto a la troponina-I y la CPK-MB. Alcanza su
pico a las 8 horas y desciende bruscamente a valores normales a las 18 horas11

Utilidad de la troponina-I, CPK-MB y mioglobina en el diagnstico del infarto de miocardio y de


los procesos de necrosis muscular de origen no cardiaco
Use of troponin-I, CPK-MB and myoglobin in the diagnosis of myocardial infarct and processes
of muscular necrosis of non-cardiac origin J.I. Ibez1, R. Sobrado1, M. Rivero2, J.M. Olite3, I.
Idoate3, I. Berrozpe1, E. Arina1, L. Metola1, J. Sesma1

1. Unidad de Urgencias.
2. Servicio de Medicina Interna.
3. Servicio de Bioqumica Clnica. Hospital Virgen del Camino Pamplona.

59.- Which of the following physical sings and symptoms is indicative of left ventricular failure?

a) Neck vein distensin


b) Ascites
c) Anorexia
d) Orthopnea

INSUFICIENCIA CARDIACA IZQUIERDA


Los sntomas ms caractersticos son la disnea paroxstica nocturna, la ortopnea y la tos. Al
examen fsico se constata la existencia de taquicardia, R3 y/o R4, pulso alternante, estertores
inspiratorios de predominio en las bases. Algunas veces se auscultan sibilancias.
Para confirmar su existencia se deben solicitar los siguientes estudios:

a. Radiografa de trax. Para constatar la existencia de cardiomegalia, vasos sanguneos


prominentes, lneas B de Kerley, (patrn en "alas de mariposa" por edema pulmonar
gravitacional) y derrame pleural.

b. Gases arteriales

c. Cuadro hemtico

d. Creatinina

e. Electrocardiograma

f. Ecocardiografa

LECTURAS RECOMENDADAS

Bigger JT. Why patients with congestive heart failure die.

Circulation 75 (suppl, IV):28, 1997

Braunwald E. Heart Disease. En: Textbook of Cardiovascular Medicine.

WB Saunders Co. Philadelphia, 1990

Matiz H. Insuficiencia cardiaca congestiva En: Diagnstico y Tratamiento Integral en Medicina.


Editado H Matiz.

Coleccin Educacin Mdica Vol. 5. Fundacin Escuela Colombiana de Medicina

Santaf de Bogot, 1991

60.- A 60-year-old woman presents to a physician complaining of a swelling in her neck. Her
past medical history is significant for rheumatoid arthritis and Sjgren syndrome. Physical
examination reveals a mildly nodular, firm, rubbery goiter. Total serum thyroxine (T4) is 10
mg/dL, and third-generation thyroid-stimulating hormone (TSH) testing shows a level of 1.2
mIU/mL. Antithyroid peroxidase antibody titers are high. Which of the following is the most likely
diagnosis?

a) Euthyroid sick syndrome


b) Graves disease
c) Hashimoto thyroiditis
d) Silent lymphocytic thyroiditis
Tiroiditis de Hashimoto
Es la causa ms frecuente de hipotiroidismo en las zonas donde no existe dficit dietario de
yodo y los sntomas de hipotiroidismo suelen ser el motivo de su diagnstico. Es frecuente su
asociacin con otras enfermedades autoinmunes.
Es caracterstica la presencia de un pequeo bocio, firme, irregular y no doloroso al tacto con
sensacin de plenitud en la garganta. Los Ac anti-TPO (anticuerpos antiperoxidasa) son
positivos en el 90% de pacientes y los antitiroglobulina en un 20 a 50%. La VSG es normal o
casi normal. La ecografa muestra una glndula hipoecognica sin ndulos.
Una vez el hipotiroidismo est establecido debe tratarse con hormona tiroidea a dosis
sustitutiva: 50 microgramos diarios inicialmente (25 si edad avanzada, enfermedades
cardiovasculares o consuntivas o niveles previos muy bajos de T4 libre) aumentando 25 mcrgr/
da cada 15 a 30 das segn los mismos criterios, hasta 75-100 micrgr/da, ajustndose
posteriormente segn controles que deben dilatarse al menos 2-3 meses desde el inicio del
tratamiento ya que las respuestas son lentas. Los pacientes con hipotiroidismo subclnico con
niveles altos de Ac anti-TPO debe ser tratados tambin por su frecuente evolucin a
hipotiroidismo clnico y mayor riesgo arteriosclertico por elevacin lipdica. Si el bocio es
grande debe administrase hormona tiroidea en dosis suficiente para frenar la TSH, lo que
puede hacer disminuir en 6 meses hasta un 30% el tamao de la glndula.
Si se observa un ndulo tiroideo debe ser estudiado mediante PAAF a fin de descartar
carcinoma tiroideo o linfoma (raro, pero con un riesgo relativo de 67 en este tipo de tiroiditis)

Lectura recomendada.
Tiroiditis de Hashimoto y sndrome de Sjgren.
Asociacin o sndrome mltiple autoinmune?
L. SIERRA SANTOS, A. SENDINO REVUELTA, R. PACHECO CUADROS,
G. APARICIO JABALQUINTO, F. J. BARBADO HERNNDE
Servicio de Medicina Interna. Hospital la Paz. Universidad Autnoma. Madrid
Sierra Santos L, Sendino Revuelta A, Pacheco Cuadros R, Aparicio Jabalquinto G, Barbado
Hernndez FJ. Tiroiditis de Hashimoto y sndrome de Sjgren. Asociacin o sndrome mltiple
autoinmune? An Med Interna (Madrid) 2001; 18: 86-87
HASHIMOTOS THYROIDITIS AND SJGRENS SYNDROME.
ASSOCIATION OR MULTIPLE AUTOIMMUNE SYNDROME?

Bibliografa:

Beers MH, Berkow R. Editores. El manual Merck. 10 ed. Madrid: Harcourt; 1999. p. 96-7.
Fauci AS, Braunwald E, Isselbacher KJ, Wilson JD, Martn JB, Kasper DL et al. Editores.
Harrison Principios de Medicina Interna. 14 ed. Madrid: McGraw Hill; 1998. p. 2310-2.
Larsen PR, Davies TF. Hipotiroidismo y tiroiditis. En: Larsen PR, Kronemberg HM, Melmed S,
Polonsky KS. Williams Tratado de Endocrinologa. 10 ed. Madrid: Elsevier; 2004. p. 465-500.
Pearce EN, Farwell AP, Braverman LE. Thyroiditis. N Engl J Med 2003; 348:2646-55

61.- El cncer de tiroides que puede producir un sndrome paraneoplsico y que se asocia a
elevaciones de calcitonina es:

a) cncer anaplsico
b) cncer de clulas de Hrttle
c) cncer papilar
d) cncer medular

El cncer medular de tiroides surge de las clulas parafoliculares de la tiroides, que


normalmente producen calcitonina. La medicin de calcitonina es importante sobre todo en el
seguimiento de los pacientes para detectar enfermedad residual o recidivante.
Jimnez RSA, Gmez VE, Bolaos GF. Tiroides. En Flores JF, Cabeza A, Calarco Z
(eds): Endocrinologa. 5 ed. Mxico. Mndez Oteo Mxico, 2005: 584-92.

62.- Se trata de paciente femenino de 43 aos diagnosticado con sndrome de Cushing al


encontrar una ACTH muy baja o suprimida, la primera posibilidad de diagnstico es:

a) enfermedad hipotlamo-hipofisiaria
b) adenoma suprarrenal
c) hiperplasia suprarrenal congnita
d) adenoma hipofisiario

Aproximadamente 20 a 25% de los pacientes con sndrome de Cushing tienen una neoplasia
suprarrenal. En la mayora de estos casos existe una produccin autnoma de cortisol por la
neoplasia, lo que conduce a descenso en los niveles de ACTH a rangos indetectables o
menores a 2 pmol/L o 10 pg/ml, por retroalimentacin negativa de la secrecin de ACTH.
Williams GH, Dluhy RG. Enfermedades de la corteza suprarrenal. En Jameson JL (ed):
Harrison. Endocrinologa. 1a ed. Madrid. MacGraw-Hill Espaa, 2006: 126-127.

63.- Masculino de 55 aos, acude a consulta por descontrol glucmico. Tiene antecedente de
DM tipo 2 de 13 aos de evolucin controlada con diferentes hipoglucemiantes orales. Desde
hace un mes est bajando de peso presenta polidipsia y poliuria a pesar de tomar su
tratamiento con metformn 850 mg 3 veces al da y glibenclamida tab 5 mg, 4 tabletas diarias,
adems de la dieta. EF: peso 68 kg, estatura 1.70, TA 140/80, FC 96x. Glucosa: 289 mg,
hemoglobina glucosilada de 11%. La conducta teraputica ms apropiada es:

a) Aumentar dosis de glibenclamida


b) Iniciar insulina de accin intermedia o prolongada
c) Iniciar insulina rpida por requerimientos
d) Aadir inhibidor de DPP IV
Dieta, ejercicio, educacin y automonitoreo

HbA1c < 7% HbA1c 7 a 8 % HbA1c > 8 %

Monoterapia con Aadir secretagogo Aadir tiazolidinediona


biguanida o tiazolidinediona o secretagogo

No metas No metas No metas

Insulina con o sin


hipoglucemiantes
Chan JL, Abrahamson MJ. Mayo Clin Proc 2003; 78:459-467.

Dieta, ejercicio, educacin y automonitoreo

HbA1c > 9%

2 hipoglucemiantes Insulina basal


Biguanida + secretagogo o preprandial o
Biguanida + Tiazolidinediona ambas
HO + insulina

Cheng YY A. CMAJ 2005; 172(2):213-26.


64.- Paciente femenino de 70 aos, acude a consulta por disnea de pequeos esfuerzos
antecedentes: diabetes mellitus en tratamiento y control mdico, infarto de miocardio hace
un ao, tiene una fraccin de eyeccin ventricular izquierda de 0,30 y est en tratamiento
habitual con aspirina, furosemida, (20 mg/da) y captopril, (25 mg/da)) exploracin fsica:
Ingurgitacin Yugular, cianosis distal en manos. TA: 140/70 mmHg, FC: 94 lpm, edema pretibial
mpi, ECG con datos de F.A.
La medida teraputica inicial en esta paciente es:

a) Iniciar el captopril por lisinopril (20 mg/da).

b) Iniciar tratamiento con digoxina (0,25 mg/da).

c) Iniciar tratamiento con espironolactona (25 mg/da).

d) Iniciar tratamiento con bisoprolol (1,25 mg/da).

Glucsicos cardiacos (digital): ICC


El tratamiento con digoxina debe aadirse a diurticos e IECAs (tratamiento triple), siempre que
no exista contraindicacin, para reforzar su eficacia cuando la respuesta teraputica en la IC
sistlica es subptima y persisten los sntomas. La digoxina est especialmente aconsejada en
pacientes con IC y fibrilacin auricular con respuesta ventricular rpida, y en la disfuncin
sistlica en clase funcional III-IV de la NYHA en ritmo sinusal, en tanto que en la disfuncin
ventricular asintomtica en fibrilacin auricular pueden ser alternativas igualmente vlidas
algunos calcioantagonistas o los betabloqueantes.
El principal efecto de la digoxina es inotrpico (slo manifiesto en presencia de IC), aunque
probablemente tambin sea vasodilatador y diurtico. Los ensayos clnicos ms
recientes19 no han podido demostrar un aumento de supervivencia en enfermos con IC
tratados con digital, aunque confirman que mejora la situacin hemodinmica y la capacidad
funcional, pudiendo resultar til para aliviar la sintomatologa, prevenir el deterioro clnico y
reducir ligeramente la morbilidad. En contrapartida, aunque no existe un aumento de las cifras
de mortalidad global, s que se ha constatado un incremento de la mortalidad de causa
arritmognica en enfermos tratados con digoxina.

65.- Se trata de masculino de 61 aos, fumador de 10 cigarrillos diarios, historia de tos y


expectoracin matutina habitual, consulta por disnea de mnimos esfuerzos y ortopnea de dos
almohadas. Exploracin fsica: TA 180/100 mmHg, presin venosa normal, auscultacin
pulmonar con crepitantes bibasales, auscultacin cardaca rtmica a 120 lpm con soplo sistlico
eyectivo I/VI en foco artico y tercer ruido. ECG: ritmo sinusal y criterios de hipertrofia
ventricular izquierda. El diagnstico ms probable es:

a) Cardiopata hipertensiva en insuficiencia cardaca.

b) Cardiopata isqumica con disfuncin sistlica.

c) Cor pulmonale crnico.

d) Insuficiencia cardaca congestiva en paciente con EPOC.


La cardiopata hipertensiva constituye la complicacin principal de la hipertensin arterial, pues
es la primera causa de morbi-mortalidad del paciente hipertenso. En la hipertensin arterial la
composicin histolgica del ventrculo izquierdo se altera globalmente, resultando lesiones que
afectan a los propios cardiomiocitos, al intersticio miocrdico y a la pared de las arterias
intramiocrdicas. En el origen del desarrollo de esas lesiones participan tanto la sobrecarga
mecnica de la pared ventricular impuesta por la presin arterial elevada, como factores
humorales sistmicos y locales que actan directamente sobre el parnquima y los vasos
miocrdicos, por ejemplo, la angiotensina II. Las consecuencias funcionales de las lesiones
estructurales miocrdicas son diversas, aunque la ms representativa es la que tiene que ver
con el desarrollo de insuficiencia cardiaca congestiva
La insuficiencia cardiaca (IC) es un sndrome clnico complejo en el que los pacientes
presentan sntomas tpicos de IC (disnea en reposo o con esfuerzo), signos tpicos de IC
(taquicardia, taquipnea, estertores pulmonares, derrame pleural, elevacin de la presin
yugular venosa, edema perifrico, hepatomegalia) y evidencia objetiva de una anomala
estructural o funcional del corazn en reposo (cardiomegalia, tercer tono, soplos cardacos,
anomalas electrocardiogrficas, concentraciones elevadas de pptidos natriurticos) (ESC,
2008).
Cardiopata hipertensiva
MARIO BENDERSKY*, DANIEL PISKORZ#, DANIEL BOCCARDO
* Profesor de Farmacologa, Facultad de Ciencias Mdicas, Universidad Nacional de Crdoba.
Unidad de Hipertensin Arterial,
Instituto Modelo de Cardiologa de Crdoba.
# Instituto de Cardiologa del Sanatorio Britnico de Rosario.
Escuela de Cardiologa, Universidad Catlica de Crdoba. Departamento de Tcnicas No
Invasivas y Arritmias, Instituto Modelo
de Cardiologa de Crdoba.
Direccin postal: Instituto Modelo de Cardiologa. Av. Sagrada Familia 359. 5003 Crdoba.
Argentina.
Sanatorio Britnico de Rosario. Paraguay 40. 2000 Rosario. Pcia. de Santa Fe. Argentina.

66.- Masculino de 64 aos, es llevado al servicio de urgencias por presentar dos horas con
dolor intenso retroesternal, que comenz en reposo, acompaado de sudoracin profusa,
nausea y vmito. Antecedentes: Tabaquismo positivo intenso. Exploracin Fsica:
auscultacin cardaca hay taquicardia y galope, Trax campos pulmonares, estertores
crepitantes en bases. Exmenes de Laboratorio: El ECG muestra ondas Q de nueva aparicin
y elevacin de S-T en DII, DIII y a VF. Elevacin de enzimas cardiacas al doble de lo normal.
El diagnstico ms probable en este paciente es:

a) Infarto anterior extenso


b) Diseccin artica con afectacin de orificios coronarios.
c) Infarto lateral
d) Infarto inferior transmural.
67.- Se trata de paciente masculino de 54 aos de edad, que se queja de debilidad muscular
desde hace aproximados 3 meses. A la E.F. Se aprecia eritema en heliotropo en su trax
superior, cuello y cara. Eritema maculopapular situado sobre nudillos de ambas manos.
El diagnstico ms probable de ste paciente es:

a) Esclerodermia
b) Dermatomiositis
c) Vasculitis
d) Artritis reumatoide

Enfermedades que se caracterizan por debilidad muscular secundaria a la inflamacin del


msculo estriado.

EPIDEMIOLOGA

INCIDENCIA: 5 casos nuevos por milln / ao


PREVALENCIA: 10 - 60 casos por milln de habitantes. Frecuencia baja.
2 PICOS DE EDAD: 5-15 aos 40-60 aos
FRECUENCIA MUJER : HOMBRE 2:1
RAZA NEGRA > ORIENTALES (4:1)

La polimiositis respeta la piel, mientras que la dermatomiositis presentar alteraciones


cutneas caractersticas acompaando a la afectacin muscular
Cuadro Clnico:
- Alteraciones musculares. Vienen marcadas por la presencia de debilidad muscular aguda
o subaguda (generalmente de inicio insidioso), simtrica y difusa, con preferencia por
musculatura proximal de extremidades (cintura plvica y escapular), tronco y cuello. En la
mayora de los casos es indoloro. Con el tiempo, desarrollan atrofia, contracturas y disminucin
de los reflejos.
- Alteraciones cutneas. La ms frecuente en la DM es una erupcin cutnea
eritematoviolcea que afecta a cuello, cara y trax. Es caracterstico tambin, el eritema
heliotropo (en prpados), que puede extenderse a otras zonas fotoexpuestas), las ppulas
de Gottron (localizadas en los nudillos), telangiectasias periungueales, a veces ulceracin
drmica y calcinosis (fundamentalmente en la DM infantil).
- Articulares. Artralgias, artritis transitorias, no erosivas, con tendencia a la simetra.
- Otras. Afectacin cardiaca variable (alteracin ECG, arritmia, miocarditis), pulmonar (fibrosis
intersticial asociada con anti Jo-1), renal (muy rara), fenmeno de Raynaud
Diagnstico:
- Analtica: aumento de VSG y de enzimas musculares, (CPK, aldolasa, GOT, GPT, LDH). La
CPK es la ms sensible y la que guarda una mejor correlacin clnica con la actividad de la
enfermedad y la valoracin de recadas. El FR es + en 20% y
ANA es + en 10-30%. Si la destruccin muscular es intensa, puede producir mioglobinuria.
- Destacan anticuerpos: anti-Jo1: en casos de PM asociado a neumonitis intersticial (sndrome
antisintetasa-miosistis, fibrosis pulmonar, artritis no erosiva y fenmeno de Raynaud).
anti-PM1 o PM-Scl: asociacin con esclerodermia.
anti-Mi, en DM.
antimioglobina.

BIBLIOGRAFA RECOMENDADA

Klippel JH, Stone JH, Crofford LJ, White PH, editors. Primer on the rheumatic diseases.
13th ed. New York: Springer-The Arthritis Foundation; 2008.

Martnez-Elizondo P, editor. Introduccin a la Reumatologa. 4a ed. Mxico: Colegio Mexicano


de Reumatologa A.C./Intersistemas S.A. de C.V.; 2008.

Firestein GS, Budd RC, Harris ED Jr, McInnes IB, Ruddy S, Sergent JS, editors. Kelleys
Textbook of Rheumatology. 8th ed. Philadelphia: Saunders Elsevier; 2009.

68.- Masculino de 50 aos que presenta sbitamente dolor intenso en primer ortejo de pie
derecho durante la noche posterior, tras ingesta de alcohol. Con los datos clnicos anteriores
usted pensara que el paciente cuenta con:

a) Hiperuricemia crnica sintomtica.


b) Sndrome de reiter.
c) Monoartritis infecciosa.
d) Artritis gotosa.

Las causas que generan hiperuricemia son mltiples, pero en general podemos dividirlo en tres
grupos:1 Hiperuricemia idioptica (10-15%) que representa los errores innatos del
metabolismo, padecimientos caracterizados por un incremento en la sntesis de purinas y
sobreproduccin de cido rico; 2 Por incremento del recambio metablico y que genera; un
exceso de cido rico srico, observado en los sndromes mieloproliferativos, neoplasias,
etctera, y 3 grupo conformado por padecimientos o condiciones que producen baja excrecin
renal de cido rico. La acidosis metablica, insuficiencia renal, enfermedades metablicas
endocrinolgicas y el uso de frmacos, son las causas ms comunes de esta situacin.
Factores de Riesgo: Todas las causas de hiperuricemia son un factor de riesgo para la gota.
1) La obesidad o el aumento o prdida repentinos de peso; 2) mayores de 40 aos; 3) sexo
masculino; 4) Miembros de la familia con gota;5) Diurticos, tales como el hidroclorotiazido; 6)
Algunas medicinas, como la aspirina; 7) Una dieta rica en cristales de nitrgeno; 8) Consumo
de alcohol; 9) Algunos tipos de cncer o tratamientos contra el cncer (por ejemplo, medicinas
citotxicas); 10) Medicinas (tales como los que combaten la apopleja y otros); 11)
Deshidratacin;12) Hipercolesterolemia;13) Enfermedad renal; 12) Desordenes endocrinos,
como el hipotiroidismo y el hiperparatiroidismo

Como es una enfermedad inflamatoria tiene un comportamiento episdico, de distribucin


universal, predomina en el hombre con el 90% de los casos y 10% en mujeres, de preferencia
posmenopusicas. De inicio ms frecuente entra la 4 y 6 dcada. Con diferentes estadios
conocidos como hiperuricemia sintomtica, artritis gotosa aguda, periodo intercrtico y gota
crnica tofcea.
Entonces, de acuerdo a los estadios de la gota podemos mencionar:
A Hiperuricemia asintomtica
B Artritis Gotosa: Es la ms comn y se caracteriza por dolor agudo, intenso, que afecta a
una ms articulaciones de predominio monoarticularlas, de acuerdo a orden de frecuencia
son; el dedo gordo del pie tobillo, tarso, rodilla y mueca.
C Periodo Intercrtico: intervalo entre un ataque y otro. El paciente queda libre de sntomas
hasta la presentacin del siguiente ataque de artritis, la duracin del periodo asintomtico es
variable. Posteriormente los periodos asintomticos se van acortando y el paciente evoluciona
hacia la cronicidad.
Gota Tofcea Crnica: Caracterizada por periodos recidivantes de artritis con depsitos de
uratos conocidos como tofos, tumoraciones sobre la articulacin que pueden estar excretando
un material calcreo.

Rev Pacea Med Fam 2006; 3(3): 2-5


Dra. Nilsa Selaya C.
Dr. Cesar Rabaza M.
Dr. Rafael Castillo R.

69.- Masculino de 65 aos con antecedentes de tabaquismo intenso desde los 20 aos de
edad, refiere disnea la cual ha presentado incremento en el ltimo mes, el examen fsico:
trax con dimensiones mayores en el dimetro antero posterior y escaso desarrollo de la masa
muscular. Hay hipersonoridad a la percusin y los sonidos pulmonares se encuentran muy
disminuidos. La Rx lateral de trax muestra aumento del aire retro esternal. Se enva para
pruebas de funcin respiratoria. El diagnstico presuntivo ms probable de ste paciente es:

a) Enfisema
b) Bronquitis crnica
c) Asma
d) Asbestosis

Un enfisema se define en trminos anatomopatolgicos por el agrandamiento permanente de


los espacios areos distales a los bronquiolos respiratorios, con destruccin de la pared
alveolar, con o sin fibrosis manifiesta.1 Es una enfermedad crnica comprendida junto con la
bronquitis crnica en la Enfermedad Pulmonar Obstructiva Crnica (EPOC).2 El nombre viene
del griego emphysema que significa "soplar el aire" o "insuflar'.3

Fisiopatologa del enfisema


Le enfermedad por sus efectos incapacita y debilita enormemente la calidad de vida del
paciente, sometindolo a una vida restringida y sedentaria. El deterioro progresivo de su
funcin pulmonar, desencadena no slo cambios fsicos en el paciente, sino tambin
alteraciones a nivel psicolgico.
El enfisematoso, se ve ceido a realizar mnimos esfuerzos. Cualquier tipo de actividad fsica,
se convierte en verdadero sufrimiento y evita realizarlas. Las ms elementales necesidades
fisiolgicas, requieren de enormes esfuerzos y gastos extras de energa.
El enfisema es una enfermedad crnica, progresiva, caracterizada por un agrandamiento
anormal y permanente de los espacios areos distales al bronquiolo terminal, acompaado de
destruccin de sus paredes sin fibrosis (8)
La caracterstica ms relevante de la enfermedad, es su limitacin al flujo areo durante los
movimientos respiratorios. Esta limitacin, medido en valores del volumen espiratorio forzado al
primer segundo (VEF 1) por debajo de los valores predictivos del paciente (< 1,0
l), ocasiona una gran morbilidad, con deterioro severo de su capacidad vital (CV).
El 25% de la resistencia total pulmonar al flujo areo, ocurre normalmente en las vas areas <
3mm, pero estos valores se aproximan al 80% en los pacientes enfisematosos
(9,10). Este flujo, est determinado por el juego entre presin retroceso pulmonar elstico.
Como el volumen pulmonar durante la expiracin disminuye, ocurre cierre prematuro de las
vas areas perifricas, debido a la prdida de elasticidad pulmonar y a la destruccin de los
alvolos que se encuentran fijados a las delgadas paredes de los bronquiolos. Al final, estos
pulmones enfisematosos son de mayores dimensiones a lo normal.
Ello se traduce en disminucin del VEF 1, y otros flujos (FEF 25-75, FEF 50), aumento del
volumen residual (VR) de la capacidad residual funcional (CRF) y de la capacidad pulmonar
total (CPT), as como, una disminucin a la prueba de difusin al monxido de carbono
(DLCO).
El atrapamiento de aire en las zonas enfisematosas, se traduce en hallazgos radiolgicos
tpicos; un pulmn hiperaireado, con escasa vasculatura pulmonar, aplanamiento de los
diafragmas y un trax ms alargado.
Al examen fsico, vemos un trax con dimensiones mayores en el dimetro antero posterior y
escaso desarrollo de la masa muscular. Hay hipersonoridad a la percusin y los sonidos
pulmonares se encuentran muy disminuidos.
Todos estos cambios en la funcin pulmonar, ocasionan desigualdad en la relacin ventilacin /
perfusin (V/Q), resultado de un incremento de espacio muerto, hipoxemia sola en fases
tempranas, acompaadas de hipercapnia en las fases terminales.
Basndose en estos datos, la indicacin quirrgica no slo se limita al pulmn enfisematoso
puro, sino tambin, al pulmn con enfermedad bulosa (11-16).

Gordon LS. Emphysema: The first two centuries and beyond. A historial overview,
with suggestions for future research: Part I. Am Rev Respir Dis 1992; 146:1334-
1344.
Gaast A, Molard-Dietmenn A, Pelletier A, Pauli G, Bieth JG. The antielastase screen of the
lower respiratory tract of alpha I-proteinase inhibitor suficiente patients with emphysema or
pneumotorax. Am Rev Respir Dis 1990;141:880-883.
Janoff A. Elastases and emphysema, current assesment of the Protease-Antiprotease
hypothesis. Am Rev Respir Dis 1985;417-433.
Silverman EK, Speizer FE. Risk factors for the development of chronic pulmonary disease. Med
Clin North Amer 1996;80:501-522.
Deslaries J. A perspective on the role of surgery in chronic obstructive lung disease.
Chest Surg Clin North Amer 1995;5:575-602.
Brantigan OC, Mller E. Surgical treatment of pulmonary emphysema. Am Surg
1957;23:789-804.
Cooper JD, Trulock EP, Triantafillou AN, Patterson GA, Pohl MS, Doloney PA, et al.
Bilateral pneumonectomy (volume reduction) for chronic pulmonary disease. J Thorac
Cardiovasc Surg 1995,109:106-119.

70.- Mujer de 62 aos con antecedentes de cncer de glndula mamaria izquierda hace 5 aos
actualmente se presenta por dolor torxico y derrame pleural. Se realiza una toracocentesis y
se analiza el fluido revelando que tiene una diferencia de protenas con el suero de 0.9 y
diferencia de DHL con suero de 1. Este fluido es un:

a) Trasudado
b) Exudado
c) Derrame
d) Infiltrado

Toracentesis y anlisis del lquido pleural. La toracentesis diagnstica requiere menos


de 30 ml de lquido. En los derrames enquistados es til la ecografa para localizar con
precisin el lquido, y con ello hacer la toracentesis ms fcil y sin riesgos (Ver gua para
drenaje y succin pleural).

Los derrames pleurales se clasifican en trasudados y exudados. Un trasudado es un filtrado de


plasma que resulta del aumento de la presin hidrostlica o de la alteracin de la permeabilidad
capilar. Los trasudados se asocian con insuficiencia cardiaca congestiva, sndrome nefrtico,
cirrosis y condiciones de sobrecarga de volumen.

El exudado es un lquido rico en protenas resultante de una inflamacin local o por una falla en
la eliminacin de protenas por los linfticos o ambos mecanismos. Los exudados se producen
en infecciones colagenopatas y neoplasias.

Existen criterios para la diferenciacin entre trasudados y exudados (los exudados deben
cumplir al menos uno de los siguientes criterios):

1. Relacin de protena pleural/srica > 0.5


2. Relacin de LDH pleural/srica > 0.6
3. LDH pleural > 200 Ul
4. Estos criterios tiene significancia diagnstica con una sensibilidad del 98% y
especificidad de 77%.

Otros criterios tiles para el diagnstico son:

a. Colesterol total en lquido pleural mayor de 60 mg/dl


b. Relacin de colesterol pleural/srico mayor 0.4

Otro anlisis del lquido pleural de importancia es la concentracin de glucosa. Un resultado


bajo (< 60 mg/dl) en el lquido pleural es sugestivo de empiema, neoplasia, TBC, LES o
pleuresa reumtica.

El pH normal es de 7.60 encontrndose menor a 7.30 en las mismas entidades patolgicas


descritas para la glucosa, y en la ruptura esofgica; en los derrames por neoplasia un pH bajo
se relaciona con menor sobrevida y menor respuesta a la pleurodesis qumica.

La medicin de triglicridos es til ante la sospecha de quilotrax (> 110 mg/dl).

La medicin de adenosina deaminasa permite la diferenciacin de TBC pleural y neoplasia


cuando es mayor de 45 UI.

El recuento y la diferenciacin celular ayudan al diagnstico etiolgico del derrame. Los


trasudados en general tienen menos de 1.000 leucocitos/ml; recuentos mayores a 10.000/ml se
ven en derrames paraneumnicos, mayores de 50.000/ml en empiema. Los derrames crnicos
(TBC, neoplasia) tienen menos de 5.000/ml. La linfocitosis es indicativa de TBC, neoplasia,
linfoma, sarcoidosis, pleuresa reumtica. Se encuentra predominio neutroflico en neumona,
embola y pancreatitis.

Ante la sospecha de neoplasia, se debe solicitar una citologa del lquido pleural, la cual tiene
una sensibilidad importante.

LECTURAS RECOMENDADAS

American College of Physicians. Diagnostic thoracentesis and pleural biopsy in pleural


effusions Ann Inter Med 103:799, 1985
Camacho Durn F, Restrepo Molina J. Enfermedades de la pleura. En: Fundamentos de
Medicina. Neumologa. Tercera Edicin.

Corporacin para Investigaciones Biolgicas. Medelln, 1986

Des Jardins T. Enfermedades pleurales En: Enfermedades Respiratorias. Terry Des Jardins
Editor.

Editorial El Manual Moderno SA Mxico DF, 1993

Light RW, Mac Gregor M. The diagnostic separation of trasudates and exudates.

Ann Intern Med 77: 507, 1972

Pacheco PM. Estudio del derrame pleural En: Enfermedades del Trax. Fidel Camacho, Jaime
Pez, Carlos Awad Editores.

Ediciones Mdicas Zambn. Santaf de Bogot, 1992

Patio JF, Arroyo de S. Gua prctica de toracentesis y de toracostoma cerrada (insercin de


tubo de trax).

Trib Mdica 89:161, 1994

Patio JF, Arroyo de S. Gua para drenaje y succin pleural. Manejo del drenaje pleural.

71.- Se trata de masculino de 44 aos que ingresa al servicio de urgencias por haber sufrido
colisin automovilstica de frente. Sufre fractura en tibia y peron, se practic ciruga ortopdica
sin complicaciones y, durante los dos primeros das del postoperatorio, el paciente estuvo
inquieto pero bien orientado. Al tercer da, el paciente se mostr de repente confuso y temeroso
y refiri visin de serpientes y araas que se desplazaban en su cama. En la exploracin se
observ midriasis, temblor de oscilaciones amplias de las manos y los prpados, sudoracin
profusa, taquicardia con latido hipercintico y temperatura de 39C. Como antecedente se
consideraba un bebedor social moderado y negaba el abuso de drogas. El diagnstico ms
probable es:

a) Delirio postanestesia.
b) Septicemia.
c) Delirio por deprivacin alcohlica.
d) Hematoma subdural.

CRITERIOS DIAGNSTICOS DE LA ASOCIACIN AMERICANA DE PSIQUIATRA PARA


LA DEPRIVACIN ALCOHLICA Y EL DELIRIO POR DEPRIVACIN ALCOHLICA (DSM-
IV)

DEPRIVACIN ALCOHLICA
A.-Cese o disminucin del uso prolongado e intenso de alcohol
B.-Dos o ms de los siguientes criterios, entre varias horas y
varios das despus de A:
1-Hiperactividad autonmica (diaforesis, taquicardia)
2-Aumento de temblor en manos
3-Insomnio
4-Nuseas y vmitos
5-Ilusiones o alucinaciones transitorias visuales, tctiles o
auditivas
6-Agitacin psicomotriz
7-Ansiedad
Crisis epilpticas tipo gran mal
C.-Agotamiento y angustia significativas o deterioro social y
ocupacional en relacin con B
D.-Exclusin de otras condiciones mdicas generales y de otros
trastornos mentales
DELIRIO POR DEPRIVACIN ALCOHLICA
A.-Alteracin del nivel de consciencia con reduccin de habilidad
para fijar, mantener o cambiar la atencin
B.-Cambio en la cognicin o desarrollo de trastorno perceptual, no
explicado por demencia preexistente
C.-Se desarrolla en horas o das y tiende a fluctuar durante el da
y tras un sndrome de deprivacin

72.- Se trata de varn de 41 aos que de manera inesperada y espontnea inicia cuadro
constituido por disnea, sensacin de ahogo o de paro respiratorio, sensacin de inestabilidad,
siente que se desmaya, palpitaciones o taquicardia, mareo, nauseas, dolor abdominal. De los
siguientes el diagnstico ms probable es:

a) Trastornos de ansiedad generalizada.


b) Trastorno fbico.
c) Trastorno de pnico.
d) Trastorno mixto ansioso-depresivo.

La caracterstica fundamental del Trastorno de Pnico es la presencia de Crisis de Pnico


recurrentes, inesperadas y que no se encuentran relacionadas con ninguna circunstancia en
particular (es decir son espontneas), sin un factor externo que las desencadene, es decir que
no son desencadenadas por una exposicin a una situacin social (como es el caso de la Fobia
Social) o frente a un objeto temido (en cuyo caso se tratara de una Fobia Especfica ). Otra
caracterstica del Trastorno de Pnico es el miedo persistente a padecer una nueva Crisis de
Pnico, esto es miedo al miedo y se lo denomina Ansiedad Anticipatoria. Esta ansiedad puede
a llegar a ser tan importante que puede llevar a confundir el diagnstico de Trastorno de Pnico
con el de Trastorno de Ansiedad Generalizada (TAG).

Estas Crisis (o Ataques) de Pnico, inesperadas y recidivantes, suelen presentar cuatro o


ms de los siguientes sntomas:

Miedo intenso a morir o a estar sufriendo un ataque cardaco o alguna enfermedad


fsica grave que ponga en riesgo la vida
Miedo intenso a volverse loco o a perder el control de si mismo
Palpitaciones (percepcin del latido cardaco) o pulsaciones aceleradas (taquicardia)
Sudoracin
Palidez
Temblores o sacudidas musculares
Sensacin de ahogo o falta de aire
Opresin en la garganta (sensacin de no poder respirar) o en el pecho
Nuseas, vmitos o molestias y dolores abdominales
Inestabilidad, mareos o desmayos
Sensacin de irrealidad (sentir al mundo externo como algo extrao)
Sensacin de no ser uno mismo (despersonalizacin)
Hormigueos (parestesias)
Escalofros o sensacin de sufrir fro intenso

Las crisis se inician bruscamente, alcanzan su mxima intensidad en los primeros diez a 15
minutos y suelen durar menos de una hora. Dejan a quien las sufre en un estado de total
agotamiento psicofsico y con un gran temor (ansiedad anticipatoria) a volver a padecer una
nueva crisis (miedo al miedo). Tambin pueden aparecer sntomas de tipo depresivo, miedo a
salir o alejarse del hogar o necesidad de hacerlo acompaado por una ser muy cercano
(familiar). La persona suele sentirse muy hipersensible y vulnerable. Este tipo de trastorno es
tan traumtico de experimentar que quien lo padece suele cambiar en forma brusca y
desfavorable sus hbitos de vida: no querer salir solo de la casa o viajar, retraimiento social,
abandono de sus actividades laborales o acadmicas.

Muchas veces la persona que se encuentra padeciendo una Crisis de Pnico tiene la
necesidad de "salir corriendo" del lugar donde se encuentra o de consultar urgentemente a un
mdico en el caso que crea que se est muriendo de verdad. Si esta crisis se experimenta por
primera vez en un lugar determinado, un tren o autobs por ejemplo, suele quedar temor de
volver a ese mismo sitio o medio de locomocin, desarrollndose de este modo una fobia al
mismo.

Un paciente que sufra de Crisis de Pnico describi su primera crisis de la siguiente forma:
"Sbitamente sent una oleada de miedo sin que hubiera razn alguna. El corazn me lata
apresuradamente, me faltaba el aire, senta que no poda respirar, el corazn lata tan fuerte
que pareca que iba a salirse por mi boca. Me dola el pecho, tena mareos, ganas de vomitar,
no poda parar de temblar. Senta que me mora. Era como una agona mortal que nunca
terminaba"

Este trastorno se lo clasifica dentro de las neurosis, no tiene nada que ver con la locura
(psicosis), ni desencadena con el tiempo en un cuadro de locura. Es importante recalcar esto
ya que muchos pacientes que sufren este trastorno, o los familiares del mismo, piensan que se
estn volviendo locos. Lo que s es frecuente es que las personas que padecen de Trastorno
de Pnico desarrollen, si no son tratados a tiempo y adecuadamente, cuadros de tipo
depresivos, fobias mltiples (especialmente agorafobia) o abuso de sustancias (alcohol o
drogas).

Referencias Bibliogrficas:

- Diagnostic and Statistical Manual of Mental Disorders, 4 edition (DSM-IV TR). American
Psychiatric Association. American Psychiatric Press, 2000.
- Kaplan and Sadock's Synopsis of Psychiatry, 9 edition. Lippincott Williams & Wilkins Press,
2003.

73.- Mujer de 34 aos acompaada de su esposo. Ella dice encontrarse muy bien, mejor que
nunca, est pudiendo hacer ms cosas e incluso se siente capaz de comprender cuestiones
muy complicadas. El marido dice que ella lleva una semana durmiendo poco y que no para; se
levanta temprano, sale a la calle, vuelve, se cambia, vuelve a salir, habla sin parar con
cualquier persona que se encuentra y no controla lo que gasta. Nunca le haba ocurrido nada
parecido y ella no acepta tener ningn problema. El diagnstico ms probable es:

a) Trastorno bipolar.
b) Trastorno psictico breve.
c) Episodio manaco.
d) Trastorno de ansiedad generalizada.
Criterios para el episodio manaco

A. Un perodo diferenciado de un estado de nimo anormal y persistentemente elevado,


expansivo o irritable, que dura al menos 1 semana (o cualquier duracin si es necesaria la
hospitalizacin).

B. Durante el perodo de alteracin del estado de nimo han persistido tres (o ms) de los
siguientes sntomas (cuatro si el estado de nimo es slo irritable) y ha habido en un grado
significativo:

1. autoestima exagerada o grandiosidad


2. Disminucin de la necesidad de dormir (p. ej., se siente descansado tras slo 3 horas de
sueo)
3. Ms hablador de lo habitual o verborreico
4. Fuga de ideas o experiencia subjetiva de que el pensamiento est acelerado
5. distraibilidad (p. ej., la atencin se desva demasiado fcilmente hacia estmulos externos
banales o irrelevantes)
6. Aumento de la actividad intencionada (ya sea socialmente, en el trabajo o los estudios, o
sexualmente) o agitacin psicomotora
7. Implicacin excesiva en actividades placenteras que tienen un alto potencial para producir
consecuencias graves (p. ej., enzarzarse en compras irrefrenables, indiscreciones sexuales o
inversiones econmicas alocadas)

C. Los sntomas no cumplen los criterios para el episodio mixto.

D. La alteracin del estado de nimo es suficientemente grave como para provocar deterioro
laboral o de las actividades sociales habituales o de las relaciones con los dems, o para
necesitar hospitalizacin con el fin de prevenir los daos a uno mismo o a los dems, o hay
sntomas psicticos.

E. Los sntomas no son debidos a los efectos fisiolgicos directos de una sustancia (p.ej. una
droga, un medicamento u otro tratamiento) ni a una enfermedad mdica (p. ej., hipertiroidismo).

Nota: Los episodios parecidos a la mana que estn claramente causados por un tratamiento
somtico antidepresivo (p. ej., un medicamento, teraputica electroconvulsiva, teraputica
lumnica) no deben ser diagnosticados como trastorno bipolar I.

74.- En un estudio transversal se tienen 1000 pacientes con cncer mamario, 32 de ellas
estaban embarazadas. A partir de estos datos, se puede concluir que:

a) El embarazo es una complicacin rara del cncer mamario


b) Si se hacen los ajustes de la edad, se puede determinar el riesgo de cncer de mama
durante el embarazo
c) En este estudio el 3.2 % de las pacientes con cncer de mama estaban embarazadas.
d) Existe asociacin causal entre estar embarazada y desarrollar cncer de mama

Los estudios transversales solo permiten recoger informacin y describir la distribucin de


frecuencias de las caractersticas de salud de la poblacin y de las posibles asociaciones de
stas con otras variables. Solo permite calcular prevalencia (32/1000X100= 3.2)

Ruiz M. A. Epidemiologa Clnica, Panamericana, 1. Ed. 2004; pg: 198


75.- Usted es un cirujano general y descubre que haciendo algunas modificaciones a una
tcnica quirrgica disminuye el tiempo de uso de quirfano, pero quiere comparar si tiene los
mismos resultados clnicos que cuando aplica la tcnica clsica, El tipo de estudio que debe
realizar es:

a) Ensayo clnico controlado


b) Casos y controles
c) Cohorte
d) Transversal

Los ensayos clnicos controlados son estrategias diseadas para evaluar la eficacia de un
tratamiento en el ser humano mediante la comparacin de la frecuencia de un determinado
evento de inters clnico o desenlace en un grupo de enfermos tratados con la terapia en
prueba con la de otro grupo de enfermos que reciben un tratamiento control.
Calva M.J.J. Estudios Clnicos Experimentales. Salud Pblica de Mxico vol.42, nm. 4, 2000
(349).

76.- Se trata de paciente femenino postoperada de colecistectoma laparoscpica, inicia con


disnea, dolor torcico y taquipnea se sospecha de tromboembolia pulmonar. El estudio de
mayor especificidad y alto grado de sensibilidad para el diagnstico es:

a) Gasometra arterial
b) Radiografa torcica
c) Electrocardiograma (ECG)
d) Arteriografa pulmonar

Allen R. M. MMS Medicina Interna. 5. Edicin. National Medical Series. Mc. Graw Hill. 2006.
(captulo 2 VIII E 6 a-c, 7, 8 a). Actualmente en caso de no encontrar en las respuestas
angiotomografa la respuesta correcta es la arteriografa pulmonar. El gammagrama
nuclear pulmonar es otra tcnica til, pero no es tan especfico como la arteriografa. Aunque
un gammagrama normal casi descarta embolia pulmonar, los resultados a menudo caen en el
lmite de probabilidad intermedio, lo que dificulta establecer un diagnstico definitivo. En la
mayor parte de los casos de embolias pulmonares, el electrocardiograma (ECG) es normal. La
desviacin aguda del eje a la derecha observado en ECG puede producir diagnstico errneo
de infarto miocrdico anterior (MI). La hipoxia, la hipocapnia y la alcalosis respiratoria son datos
clsicos de la gasometra, pero son inespecficos de embolia pulmonar. La radiografa de trax
es normal, sobre todo si no ha habido infarto.

77.-Femenino de 23 aos refiere que lleva un tiempo con menos apetito y durmiendo menos
ya que le cuesta poder dormirse. Es una buena estudiante, pero en la ltima evaluacin le han
quedado cuatro asignaturas y le cuesta concentrarse en los estudios. Los dos ltimos fines de
semana no ha salido con sus amigas porque no le apeteca y se siente irritada con su familia,
aunque no entiende porqu. Tiene dolores de cabeza y a veces le viene la idea de la muerte a
la imaginacin aunque piensa que no lo hara por sus sentimientos religiosos. Nunca le haba
sucedido algo parecido. El diagnstico ms probable es:

a) Trastorno depresivo mayor.

b) Trastorno ciclotmico.

c) Anorexia.

d) Episodio depresivo mayor.

Criterios para el diagnstico del episodio depresivo mayor (DSM-IV)


A. Presencia de cinco (o ms) de los siguientes sntomas durante un perodo de 2 semanas,
que representan un cambio respecto a la actividad previa; uno de los sntomas debe ser (1)
estado de nimo depresivo o (2) prdida de inters o de la capacidad para el placer.

Nota: No incluir los sntomas que son claramente debidos a enfermedad mdica o las ideas
delirantes o alucinaciones no congruentes con el estado de nimo.

1. estado de nimo depresivo la mayor parte del da, casi cada da segn lo indica el propio
sujeto (p. ej., se siente triste o vaco) o la observacin realizada por otros (p. ej., llanto). Nota:
En los nios y adolescentes el estado de nimo puede ser irritable
2. disminucin acusada del inters o de la capacidad para el placer en todas o casi todas las
actividades, la mayor parte del da, casi cada da (segn refiere el propio sujeto u observan los
dems)
3. prdida importante de peso sin hacer rgimen o aumento de peso (p. ej., un cambio de ms
del 5 % del peso corporal en 1 mes), o prdida o aumento del apetito casi cada da. Nota: En
nios hay que valorar el fracaso en lograr los aumentos de peso esperables
4. insomnio o hipersomnia casi cada da
5. agitacin o enlentecimiento psicomotores casi cada da (observable por los dems, no meras
sensaciones de inquietud o de estar enlentecido)
6. fatiga o prdida de energa casi cada da
7. sentimientos de inutilidad o de culpa excesivos o inapropiados (que pueden ser delirantes)
casi cada da (no los simples autorreproches o culpabilidad por el hecho de estar enfermo)
8. disminucin de la capacidad para pensar o concentrarse, o indecisin, casi cada da (ya sea
una atribucin subjetiva o una observacin ajena)
9. pensamientos recurrentes de muerte (no slo temor a la muerte), ideacin suicida recurrente
sin un plan especfico o una tentativa de suicidio o un plan especfico para suicidarse
B. Los sntomas no cumplen los criterios para un episodio mixto.

C. Los sntomas provocan malestar clnicamente significativo o deterioro social, laboral o de


otras reas importantes de la actividad del individuo.

D. Los sntomas no son debidos a los efectos fisiolgicos directos de una sustancia (p. ej., una
droga, un medicamento) o una enfermedad mdica (p. ej., hipotiroidismo).

E. Los sntomas no se explican mejor por la presencia de un duelo (p. ej., despus de la
prdida de un ser querido), los sntomas persisten durante ms de 2 meses o se caracterizan
por una acusada incapacidad funcional, preocupaciones mrbidas de inutilidad, ideacin
suicida, sntomas psicticos o enlentecimiento psicomotor.

Bibliografa:

1. DSM-IV. American Psychiatric Association. . Diagnostic and Statistical Manual of Mental


Disorders (4th Ed.). Washington, DC.

78.- The Virchow triad of thrombus in the etiology of pulmonary embolism is constituted by:

a) b) Venous stasis, endothelial injury and hypoxemia.


b) Hypoxemia, hypercoagulability and cough
c) Venous stasis, hypercoagulability, and endothelial damage.
d) Dry Cough

Tromboembolismo pulmonar

Etiologa:
El 90% de los casos de tromboembolismo pulmonar tienen su origen en venas de las
extremidades. Es favorecida por la triada de Virchow: stasis venosa, dao de la ntima,
aumento de la coagulabilidad (puerperio, ciruga mayor, cncer, uso de anticonceptivos,
policitemia vera, sndrome de hipercoagulabilidad (deficiencia de PC, PS, ATIII, resistencia a la
protena C).

Fisiopatologa.

Efectos respiratorios: aumento del espacio muerto alveolar, broncoconstriccin,


taquipnea e hipoxemia, alteracin ventilacin/perfusin (V/Q) por redistribucin de flujo.

Efectos hemodinmicos: La reduccin mecnica leve o moderada no produce


aumento de la resistencia en forma significativa. Sobre el 50% de los casos presentan
incremento brusco de la resistencia y presin. Al efecto del mbolo se suman las aminas
liberadas por las plaquetas. En el 60-70% se desencadena cor pulmonale agudo con
disminucin brusca del gasto cardiaco.

Manifestaciones clnicas del tromboembolismo pulmonar:

Disnea, Taquipnea, Dolor pleurtico, Crepitantes, Tos, Taquicardia, Hemoptisis,


broncoespasmo, cianosis, sncope, palpitaciones. Sndromes: disnea aguda de causa
desconocida, hemoptisis y/o dolor pleurtico, shock cardiognico.

British Thoracic Society, Standards of Care Committee. Suspected acute pulmonary


embolism: A practical approach. Thorax 1.997; 52 (suppl 4): S1-S23.

The PIOPED Investigators. Value of the ventilation/perfusion scan in acute


pulmonary embolism: results of the Prospective Investigation of Pulmonary
Embolism (PIOPED). JAMA 1.990; 263: 2.753-9.

Goodman PC. Spiral CT for pulmonary embolism Sem Resp Crit Care Med 2.000;
21(6): 503-10.

79.- A 27-year-old woman has been sad for the last two weeks. She is fatigued a has a hard
time concentrating at work. Just a few weeks earlier she was energetic an enthusiastic, and was
able to work 10-12 hours a day with little sleep and go dancing at night. Her husband wants a
divorce because he is tired of these constant ups and downs. The most accurate diagnosis is:

a) Borderline personality disorder


b) Seasonal mood disorder
c) Dissociative identity disorder
d) Cyclothymic disorder

Criterios para el diagnstico de


F34.0 Trastorno ciclotmico (301.13)

A. Presencia, durante al menos 2 aos, de numerosos perodos de sntomas hipomanacos y


numerosos perodos de sntomas depresivo que no cumplen los criterios para un episodio
depresivo mayor.

Nota: En los nios y adolescentes la duracin debe ser de al menos 1 ao.


B. Durante el perodo de ms de 2 aos (1 ao en nios y adolescentes) la persona no ha
dejado de presentar los sntomas del Criterio A durante un tiempo superior a los 2 meses.

C. Durante los primeros 2 aos de la alteracin no se ha presentado ningn episodio depresivo


mayor, episodio manaco o episodio mixto.

Nota: Despus de los 2 aos iniciales del trastorno ciclotmico (1 ao en los nios y
adolescentes), puede haber episodios manacos o mixtos superpuestos al trastorno ciclotmico
(en cuyo caso se diagnostican ambos trastornos, el ciclotmico y el trastorno bipolar I) o
episodios depresivos mayores (en cuyo caso se diagnostican ambos trastornos, el ciclotmico y
el trastorno bipolar II).

D. Los sntomas del Criterio A no se explican mejor por la presencia de un trastorno


esquizoafectivo y no estn superpuestos a una esquizofrenia, un trastorno esquizofreniforme,
un trastorno delirante o un trastorno psictico no especificado.

E. Los sntomas no son debidos a los efectos fisiolgicos directos de una sustancia (p. ej., una
droga, un medicamento) o a una enfermedad mdica (p. ej., hipertiroidismo).

F. Los sntomas provocan malestar clnicamente significativo o deterioro social, laboral o de


otras reas importantes de la actividad del individuo.

80.- Se trata de masculino de 29 aos, acude al servicio de Urgencias por cefalea intensa de
12 horas de evolucin. Al interrogatorio dirigido refiere debilidad muscular desde hace 4 meses.
Exploracin fsica: TA: 170/110, FC:96 x. Estatura: 1.75 cm, peso 80 kg. Laboratorio: glucosa
72 mg/dL, BUN 4.5 mg/dl, creatinina 0.72 mg/dl, Na: 142 mq/l, K: 2.1 mEq/l.
La causa ms frecuente de la alteracin que presenta el paciente es:

a) un tumor productor de catecolaminas.


b) un aumento en la endotelina 1
c) aumento en la produccin de aldosterona por hiperplasia o adenoma.
d) resistencia a la insulina
HIPERALDOSTERONISMO
CRITERIOS DE DIAGNSTICO

Hipertensin arterial
Actividad de renina baja
Aldosterona alta y no se inhibe con
expansin de volumen

Tx: ADENOMA.. QUIRRGICO


HIPERPLASIA ESPIRONOLACTONA

El hiperaldosteronismo constituye la forma ms prevalente de hipertensin arterial secundaria.


Adems el hiperaldosteronismo ha cobrado relevancia al demostrarse que la aldosterona per
se puede ejercer un efecto deletreo directo en varios rganos, independiente del aumento de
la presin arterial. El screeningdebera realizarse en hipertensos moderados, severos o
resistentes a terapia usando la relacin aldosterona/renina y no la medicin del potasio
plasmtico. La sospecha de hiperaldosteronismo debe certificarse usando tests confirmatorios
(fludrocortisona o sobrecarga salina). Una vez confirmada su existencia deben usarse
procedimientos que permitan su clasificacin para orientar la terapia (mdica o quirrgica). La
deteccin de formas hereditarias de la enfermedad abre un campo de investigacin que no slo
permitir entender mejor la fisiopatologa de la enfermedad sino que tambin permitir a futuro
el consejo gentico en familias afectadas.

REFERENCIAS

1. Kearney PM, Whelton M, Reynolds K, Muntner P, Whelton PK, He J. Global burden of


hypertension: analysis of worldwide data. Lancet 2005; 365: 217-23.

2. Jackson RV, Lafferty A, Torpy DJ, Stratakis C. New genetic insights in familial
hyperaldosteronism. Ann N YAcad Sci 2002; 970: 77-88

3. Conn JW, Cohen EL, Rovner DR, Nesbit RM. Nor-mokalemic Primary Aldosteronism. A
Detectable Cause of Curable "Essential" Hypertension. JAMA 1965; 193: 200-6.

4. Tucker RM, Labarthe DR. Frequency of surgical treatment for hypertension in adults at the
Mayo Clinic from 1973 through 1975. Mayo Clin Proc 1977; 52: 549-5.

5. Blumenfeld JD, Sealey JE, Schlussel Y, Vaughan ED Jr, Sos TA, Atlas SA et al. Diagnosis
and treatment of primary hyperaldosteronism. Ann Intern Med 1994; 121: 877-85.

6. Mulatero P, Stowasser M, Loh KC, Fardella CE, Gordon RD, Mosso L et al. Increased
diagnosis of primary aldosteronism, including surgically correctable forms, in centers from five
continents. J Clin Endocrinol Metab 2004; 89: 1045-50.

You might also like